You are on page 1of 76

JEE (MAIN+ADVANCED)

LIMIT, CONTINUITY &


DIFFERENTIABILITY OF
FUNCTION
CONTENT

S.No Pages

1. Theory 01 – 21

2. Exercise-1 (Special DPP) 22 – 45

3. Exercise-2 46 – 51

4. Exercise-3 (Section-A) 52 – 56
[Previous years JEE-Main problems]

5. Exercise-3 (Section-B) 56 – 62
[Previous years JEE-Advanced problems]

6. Exercise-4 (Section-A) 63 – 64
[Previous years CBSE problems]

7. Exercise-4 (Section-B) 64 – 65
[Potential Problems for Board Preparations]

8. Exercise-5 (Rank Booster) 66 – 67

9. Answer Key 68 – 71
LIMIT, CONTINUITY AND DIFFERENTIABILITY OF FUNCTION

LIMIT

1. CONCEPT OF LIMITS :
Suppose f(x) is a real-valued function and c is a real number. The expression Lim f(x) = L means that
x c
f(x) can be as close to L as desired by making x sufficiently close to c. In such a case, we say that limit
of f, as x approaches c, is L. Note that this statement is true even if f(c)  L. Indeed, the function
f(x) need not even be defined at c. Two examples help illustrate this.
Consider f(x) = x – 1 as x approaches 2. In this case, f (x) is defined at 2, and it equals its limiting value 1.

f (1.9) f (1.99) f (1.999) f ( 2) f (2.001) f (2.01) f (2.1)


0 .9 0.99 0.999 1 1.001 1.01 1 .1

As x approaches 2, f(x) approaches 1 and hence we have lim f ( x )  1 .


x 2
x 4 2
f(x) = in this case x approaches 2 the limiting value of f (x) is equal to 4 even if f (x) is not
x2
defined at x = 2.

f (1.9) f (1.99) f (1.999) f ( 2 .0 ) f (2.001) f (2.01) f (2.1)


3 .9 3.99 3.999  undefined  4.001 4.01 4.10

Thus, f (x) can be made arbitrarily close to the limit of 4 just by making x sufficiently close to 2.
There are two ways in which one can approach in a number c either from right or left.
This leads to two limits :
(i) Right hand limit (RHL) (ii) Left hand limit (LHL)
Left-hand and Right-Hand Limits :
Right hand limit of a function is that value of f (x) which function tends to as x moves from right on the
number line to number 'a', that is, RHL = Lim f ( x ) = Lim f (a  h ) (where h > 0)
x a h 0

Left hand limit of a function is that value of f (x) which function tends to, as x moves from left on the
number line to number 'a'. That is, LHL = Lim f ( x ) = Lim f (a  h ) (where h > 0)
x a h 0

Existence of Limit :

If follows from the discussions made in the previous two sections that lim f (x) exists if both lim f (x)
x a x a
and lim f (x) exist and are equal.
x a

Thus, lim f (x) exists  lim f (x) = lim f (x).


x a x a x a
f (a–) = lim f (a  h )
h 0

f (a )  lim f (a  h )
h 0
lim f ( x ) exists  f (a–) = f (a+) [RHL = LHL]
xa

BANSAL CLASSES Private Ltd. ‘Bansal Tower’, A-10, Road No.-1, I.P.I.A., Kota-05 Page # 1
LIMIT, CONTINUITY AND DIFFERENTIABILITY OF FUNCTION

One sided limits :

Let the function f (x) be defined in x  [a, b]. Sometimes we need to calculate lim f (x) or lim f (x).
x b x a

In such case lim f (x) = lim f ( x ) = RHL at x = a, as there is no left neighbourhood of x = a in the
x a x a
domain of f(x).
Similarly lim f (x) = lim f (x) = LHL at x = b. As there is no right neighbourhood of x = b in the
x b x b
domain of f (x).
For example, f (x) = cos–1 x
lim f ( x )  lim cos1 x  0
x 1 x 1

lim f ( x )  lim cos1 x  


x 1 x 1

THE ALGEBRA OF LIMITS :

Let lim f (x) = l and lim g (x) = m. If l and m exist, then


x a x a

1. lim (f ± g) (x) = lim f (x) ± lim g (x) = l ± m


x a x a x a

2. lim (fg) (x) = lim f (x) lim g (x) = l m


x a x a x a

f  lim f ( x ) l
lim  ( x )  a
x
 , provided m  0
3. x a 
g lim g( x ) m
x a

4. lim k f(x) = k. lim f(x), where k is a constant


x a x a

5. lim f ( x )  lim f ( x )  | l |
x a x a

lim g ( x )
6. lim f ( x ) g ( x )  lim f ( x ) xa  lm , l < 0
x a x a

7. lim fog( x )  f  lim g( x )   f (m) , only if f is continuous at g(x) = m


x a  x a 
In particular,

(a) lim log f ( x )  log lim f ( x )   log l


lim f ( x )
(b) lim ef ( x )  e xa  el
x a  x a  x a

1
8. If lim f (x) = +  or – , then lim = 0.
x a x a f (x)

9. If f(x)  g(x) for every x in the NBD of a, then lim f(x)  lim g (x).
x a x a

BANSAL CLASSES Private Ltd. ‘Bansal Tower’, A-10, Road No.-1, I.P.I.A., Kota-05 Page # 2
LIMIT, CONTINUITY AND DIFFERENTIABILITY OF FUNCTION

POINTS TO REMEMBER :

(1) If lim f(x) g(x) exists, then we can have the following cases :
x c

(a) Both lim f(x) and lim g(x) exist. Obviously, then lim f(x) g(x) exists.
x c x c x c

(b) lim f(x) exists and lim g(x) does not exist, then also lim f (x) g (x) can exist.
x c x c x c
1
Consider f(x) = x; g(x) = , now lim f(x) · g(x) exists = 1.Also lim f(x) = 0 exists but lim g(x) does
x x 0 x 0 x 0
not exist.
(c) Both lim f(x) and lim g(x) do not exist.
x c x c

1 if x  0 2 if x  0
Let f be defined as f(x) =  . Let g(x) = 1 if x  0 .
2 if x  0 
Then f(x) g(x) = 2, and so lim f(x) × g(x) exists, while lim f(x) and lim g(x) do not exist.
x 0 x 0 x 0

(2) If lim [f(x) + g(x)] exists then we can have the following cases :
x c

(a) If lim f(x) exists, then lim g(x) must exist.


x c x c
Proof : This is true as g = (f + g) – f.
Therefore, by the limit theorem, lim g(x) = lim (f(x) + g(x)) – lim f(x) which exists.
x c x c x 0
(b) Both lim f(x) and lim g(x) do not exist.
x c x c

Consider lim [x] and lim {x}, where [·] and {·} represent greatest integer and fractional part function,
x 1 x 1
respectively. Here both the limits do not exist but Lim x   {x}  lim x = 1 exists.
x 1 x 1
For Example :
1 1
(a) f (x) = and g (x) = at x = 0 Lim f ( x )  DNE , Lim g ( x )  DNE (Dose not exist)
sin x tan x x0 x0

x
1  cos x 2 sin 2
 1 1  2 = Lim tan x  0 (exist).
But Lim    = 0 = Lim = Lim
x 0  sin x tan x  x  0 sin x x0 x x x0 2
2 sin cos
2 2
(b) f (x) = sgn x and g (x) = [x] then Lim sgn x  [ x ] does not exist.
x 0

as Lim sgn x  [ x ]  1 , Lim sgn x  [ x ]  2


x 0  x 0 

while Lim f ( x )  DNE , Lim g ( x )  DNE .


x0 x0
(c) f (x) = [x] and g (x) = {x}; F (x) = [x] {x}
Lim [ x ] ·{x} does not exist but Lim [ x ]{x} exist and is equal to zero.
x0 x1

(d) If f (x) = e[x] ; g(x) = e{x} then Lim e[ x ] · e{x}  Lim e x which exists.
x 0 x 0

BANSAL CLASSES Private Ltd. ‘Bansal Tower’, A-10, Road No.-1, I.P.I.A., Kota-05 Page # 3
LIMIT, CONTINUITY AND DIFFERENTIABILITY OF FUNCTION

INDETERMINANT FORMS :
0 
Indeterminant forms are , ,  –  , 0 ×  , 1 , 00 and 0
0 

EVALUATION OF ALGEBRAIC LIMITS :

(i) Direct Substitution Method :


( x )
Consider the following limits : (i) lim f(x) (ii) lim
x a x a  ( x )
 (a )
If f(a) and exist and are fixed real numbers and (a)  0 then we say that lim f(x) = f(a) and
 (a ) x a

 ( x )  (a )
lim  .
x a  ( x )  (a )

(ii) Fractorization Method :

f (x)
Consider lim
xa g ( x )

f (x ) 0
If by substituting x = a, reduces to the form , then (x – a) is a factor of both f(x) and g(x). So,
g( x ) 0
we first factorize f(x) and g(x) and then cancel out the common factor to evaluate the limit.

(iii) Rationalization Method :


This is particularlyused when either the numerator or the denominator or both involve expression consists
0 
of squares roots and on substituting the value of x the rational expression takes the form , .
0 

(iv) Evaluation of Algebraic Limit Using Some Standard Limits :

Recall the binomial expansion for any rational power


n (n  1) 2 n (n  1)(n  2) 3
(1 + x)n = 1 + nx + x  x 
2! 3!
where | x | < 1
When x is infinitely small (approaching to zero) such that we can ignore higher powers of x, then we have
(1 + x)n = 1 + nx (approximately).
Following theorem will be used to evaluate some algebraic limits :
xn  an
Theorem : If n  Q, then lim = nan–1
x a x  a

BANSAL CLASSES Private Ltd. ‘Bansal Tower’, A-10, Road No.-1, I.P.I.A., Kota-05 Page # 4
LIMIT, CONTINUITY AND DIFFERENTIABILITY OF FUNCTION

(v) Evaluation of Algebraic Limits at Infinity :


1 1
We know that lim = 0 and lim 2 = 0
x   x x  x

1
 lim f ( x )  lim f  
x  y 0  y 

S A N DW I CH TH E OR E M OR S Q U E E Z E P LA Y TH E OR E M F OR
EVALUTATING LIMITS :
General: The squeeze principle is used on limit problems where the usual algebraic methods (factorisation
or algebraic manipulation etc.) are not effective. However it requires to “squeeze” our problem
in between two other simpler function whose limits can be easily computed and equal. Use of
Squeeze principle requires accurate analysis, indepth algebra skills and careful use of inequalities.

Statement: If f , g and h are 3 functions such that f (x)  g (x) < h (x) for all x in some interval containing the
point x =c, and if
Lim f (x) = Lim h(x) = L  Lim g (x) = L
x c x c x c

From the figure note that Lim g (x) = 1.


x 0

Note: (i) the quantity c may be a finite number, +  or – .


Similarly Lmay be finite number, +  or – .

EVALUATION OF TRIGONOMETRIC LIMITS :


sin 
(i) Lim  1 (where  is in radians)
 0 

Proof : Consider a circle of radius r. Let O be the centre of the circle P


B
such that AOB =  where  is measured in radians and it is very small.
Suppose the tangent at A meets OB produced at P. From figure, we
have  A
O r
Area of OAB < Area of sector OAB < Area of OAP
1 1 1
 OA × OB sin  < (OA)2  < OA × AP
2 2 2
1 2 1 1
 r sin  < r2  < r2 tan  [In OAP, AP = OA tan ]
2 2 2
sin 
 sin  <  < tan   1> > cos 

sin  sin 
 1  lim  lim cos  or, lim cos   lim 1
 0   0  0  0 

sin  sin 
 1  lim 1  lim 1 (By Sandwich Theorem)
 0   0 

BANSAL CLASSES Private Ltd. ‘Bansal Tower’, A-10, Road No.-1, I.P.I.A., Kota-05 Page # 5
LIMIT, CONTINUITY AND DIFFERENTIABILITY OF FUNCTION

sin 
(ii) lim 1
 0 

tan  sin  1 sin  1


We have lim  lim  lim · lim 1
 0  0  cos   0  0 cos 

sin(  a )
(iii) lim 1
 a a
sin(  a ) sin(a  h  a )  lim sin h  1
We have lim  lim
 a a h 0 ( a  h  a ) h 0 h

tan(  a ) sin 1 x
(iv) lim 1 (v) lim 1
 a a  0 x

tan 1 x 1  cos x 1
 (remember).
(vi) lim 1 (vii) Lim
x 0 x x x2 2

Note : Let [ · ] denotes greatest integer function


 sin x   sin x   tan x 
Lim  0  xLim 1 Lim  1
x0  x  0 x  x0  x 
(i) (ii) (iii)
  

 sin 1 x   x 
(iv) Lim   1 (v) Lim  1   0
x0
 x  x  0  sin x 

EVALUATION OF EXPONENTIAL AND LOGARITHMIC LIMITS :

In order to evaluate these type of limit, we use the following standard results.
a x 1
1. lim  log e a
x 0 x

ex  1
2. lim = 1 (replace a by e in the above proof)
x 0 x

log(1  x )
3. lim 1
x 0 x

LIMITS OF THE FORM Lim (f (x))g(x) :


x a
Form : 1
x
1
 1
1. lim (1  x ) x  e or lim 1    e
x 0 x   x
2. If lim f ( x ) = 1 and lim g ( x ) = 
xa xa

BANSAL CLASSES Private Ltd. ‘Bansal Tower’, A-10, Road No.-1, I.P.I.A., Kota-05 Page # 6
LIMIT, CONTINUITY AND DIFFERENTIABILITY OF FUNCTION

Then L = xlim f ( x )g ( x )
a
1 ( f ( x ) 1)g ( x )
 lim (1  (f ( x )  1)) f ( x )1
x a

lim ( f ( x ) 1)g ( x )
  1
  x a lim ( f ( x ) 1)g ( x )
  lim  (1  (f ( x )  1)) f ( x )1 
 x a    e x a

LIMITS OF FUNCTIONS HAVING BUILT IN LIMIT WITH THEM :


Examples :
0, 0  a 1 , 0  a 1
 n
Lim a  1,
n
a 1 Lim
, n a  1, a 1 .
n 0, a 1
, a 1

EXPANSION OF SOME IMPORTANT FUNCTIONS:

Expansion of function like Binomial expansion, exponential & logarithmic expansion, expansion of sinx ,
cosx , tanx should be remembered by heart & are given below :

x ln a x 2 ln 2 a x 3ln 3a
(i) ax  1    .........a  0
1! 2! 3!
x x 2 x3
(ii) e  1 
x
  ............  x  R
1! 2! 3!
x 2 x3 x 4
(iii) ln(1+ x) = x     .........for  1  x  1
2 3 4
x3 x5 x7   
(iv) sin x  x     .......    x  
3! 5! 7!  2 2

x2 x4 x6   
(v) cos x  1     ...... x    , 
2! 4! 6!  2 2

x 3 2x 5   
(vi) tan x= x    ........ x    , 
3 15  2 2

x3 x5 x7
(vii) tan–1x = x     .......
3 5 7
12 3 12.32 5 12.32.52 7
(viii) sin–1x = x x  x  x  .......
3! 5! 7!
x 2 5x 4 61x 6
(ix) sec–1x =1     ......
2! 4! 6!

BANSAL CLASSES Private Ltd. ‘Bansal Tower’, A-10, Road No.-1, I.P.I.A., Kota-05 Page # 7
LIMIT, CONTINUITY AND DIFFERENTIABILITY OF FUNCTION

CONTINUITY
1. GENERAL INTRODUCTION :
After conceiving the notion of limits the next element which is taken into consideration is the continuity of
function. Qualitativelythe graph of a function is said to be continuous at x = a if while travelling along the
graph of the function and in crossing over the point at x = a either from Left to Right or from Right to Left
one does not have to lift his pen. In case one has to lift his pen the graph of the function is said to have a
break or discontinuity at x = a. Different type of situations which may come up at x = a along the graph
can be :

Figure (2) – (6) is discontinuous at x = a and in figure (1) f is continuous at x = a

2. DEFINITION OF CONTINUITY OF A FUNCTION :


A function f(x) is said to be continuous at x = a,
if Lim f(x) = f (a).
x a
 Lim f(a – h)= Lim f (a + h) = f (a) = a finite quantity..
h 0 h 0
i.e. LHL at (x = a) = RHL at (x = a) = value of f (x) at (x = a) = a finite quantity.
Note:
(i) Continuity at x = a  existence of limit at x = a, but not the converse
(ii) Continuity at x = a  f is well defined at x = a, but not the converse
(iii) Discontinuity at x = a is meaningful to talk if in the immediate neighbourhood of
x = a, i.e. the function has a graph in the immediate neighbourhood of x = a, not necessarily at x = a.
1 1
(iv) Continuity is always w.r.t. in the domain of function and hence f (x) = , , tan x are all continuous
x 1 x
1 1
functions but if you want to talk about discontinuity then we can say is discontinuous at x = 1,
x 1 x
is discontinuous at x = 0.
Note that all rational functions are continuous. Because continuity is always talked about in the domain
of f (x).
(v) Point function are continuous.
e.g. f (x) = 1  x  x  1 , f (x) = x  x
BANSAL CLASSES Private Ltd. ‘Bansal Tower’, A-10, Road No.-1, I.P.I.A., Kota-05 Page # 8
LIMIT, CONTINUITY AND DIFFERENTIABILITY OF FUNCTION

(vi) Inverse of a discontinuous function can be continuous.


1  x 2 if x  0
e.g. f (x) = 0 if x  0 is discontinuous at x = 0 but its inverse function

 (1  x ) if x  0
2

 x  1 if x  1
f –1(x) = 0 if x  0 which is a continuous function and its graph is as shown.

  (1  x ) if x  1

f (x) 1+x2 f –1 (x)

1
–1
x x
0 0 1
–1
–(1+x2)

3. CONTINUITY IN AN INTERVAL :
(a) A function 'f 'is said tobe continuous in (a, b) if 'f 'is continuous at each& everypoint (a,b).
(b) A function 'f' is said to be continuous in a closed interval  a , b if :
(i) 'f' is continuous in the open interval (a , b) &
(ii) RHL exists at ‘a’ i.e. Lim f(x) = f(a) = a finite quantity .
x a 
(iii) LHL exists at ‘b’ i.e. Lim f(x) = f(b) = a finite quantity .
x b

4. REASONS OF DISCONTINUITY :
A function can be discontinuous due to the following reasons.
(i) Lim f (x) does not exist. ( f (a) may or may not be defined)
x a
i.e. Lim f (a + h)  Lim f (a – h)
h 0 h 0
e.g. f (x) = [x] discontinuous at all integer points f (x) = sgn x discontinuous at x = 0
x
f (x) = discontinuous at x = 1.
x 1
(ii) Lim f (x) exists but is not equal to f (a) i.e. Lim f (a + h) = Lim f (a – h)  f (a)
x a h 0 h 0
x
(1  x ) tan if x  1
f (x) =  2
 
if x  1
2

 x  h
Lim f ( x )  Lim 1  x  tan   = Lim
2

x 1 x 1  2  h 1  h  
tan  
 2 
 Lim f ( x )  f (1)  f(x) is discontinuous at x = 1.
x 1

BANSAL CLASSES Private Ltd. ‘Bansal Tower’, A-10, Road No.-1, I.P.I.A., Kota-05 Page # 9
LIMIT, CONTINUITY AND DIFFERENTIABILITY OF FUNCTION

(iii) f (a) is not defined


1
eg. f (x) = x  1
To understand explicitly the reasons of discontinuity. Consider the following graph of a function. Note
that

(a) f is continuous at x = 0 and x = 4

(b) f is discontinuous at x = 1 as limit does not


exist

(c) f is discontinuous at x = 2 as f (2) is not


defined althoughlimit exist.
O
(d) f is discontinuous at x = 3 as
Lim f (x)  f (3)
x 3

(e) f is discontinuous at x = 5 as neither the


limit exist nor f is defined at x = 5

Note:
(i) Every polynomial function is continous at every point of the real line.
f (x) = a0xn + a1xn–1 + a2xn–2 + …… + an  x  R
(ii) Every rational function is continuous at every point where its denominator is different from zero.
(iii) Logarithmic functions, exponential functions, trigonometric functions, inverse circular functions, and
modulus functions are continuous in their domain.

5. TYPES OF DISCONTINUITIES :

Types of discontinuity

Removable discontinuity Non-Removable discontinuity

(A) (B) (A) (B) (C)


Missing point Isolated point Finite type Infinite type Oscillatory
discontinuity discontinuity

5.1 REMOVABLE DISCONTINUITY :


Here lim f ( x ) necessarily exists, but is either not equal to f (a) or f (a) is not defined. In this case,
xa

therefore it is possible to redefine the function in such a manner that Lim f ( x )  f (a ) and thus making
x a

the function continuous. These discontinuities can be further classified as

BANSAL CLASSES Private Ltd. ‘Bansal Tower’, A-10, Road No.-1, I.P.I.A., Kota-05 Page # 10
LIMIT, CONTINUITY AND DIFFERENTIABILITY OF FUNCTION

(A) Missing point discontinuity :


Here Lim f ( x ) exists. But f(a) is not defined.
xa

( x  1)(9  x 2 )
(a) f(x)= x1
x 1

at x = 1, f(1) is not defined. Hence f(x) has missing point of discontinuity at x = 1.

x2  4
(b) f (x)= x  2
x2

f(2) is not defined. Hence, f(x) has missing point of discontinuity at x = 2.

sin x
(c) f (x)= ,x0
x

f(0) is not defined. f(x) has missing point of discontinuity at x = 0.

(B) Isolated point discontinuity :

Here Lim f ( x ) exists, also f(a) is defined but Lim f ( x )  f (a )


xa xa

 0 if x  I
(a) f (x) = [x]+[–x] = 
  1 if x  I

has isolated point of discontinuity at all integral points.


0 if x  2n 
 2
(b) f(x)=sgn(cos2x–2sinx+3) = sgn (2(2 + sin x)(1 – sinx)) = 
 
 1 if x  2n 
2


has an isolated point at x = 0 discontinuity as x=2n+
2

BANSAL CLASSES Private Ltd. ‘Bansal Tower’, A-10, Road No.-1, I.P.I.A., Kota-05 Page # 11
LIMIT, CONTINUITY AND DIFFERENTIABILITY OF FUNCTION

5.2 NON-REMOVABLE DISCONTINUITY :

Here Lim f ( x ) does not exists and therefore it is not possible to redefine the function in anymanner to
xa
make it continuous. Such discontinuities can be further classified into 3 fold.
(a) Finite type (both limits finite and unequal)

1  1 
f (0  ) 
(i) Lim tan   2 ; jump =
x 0 x  
f (0 )  
2

| sin x | f (0  )  1
(ii) Lim ; jump = 2
x 
f (0 )  1
x0

f (2  )  1
[x ] 1
(iii) Lim ; jump =
x 2 x 1 2
f (2  ) 
2
In this case non negative difference between the two limits is called the Jump of discontinuity.Afunction
having a finite number of jumps in a given interval I is called a Piece Wise Continuous or Sectionally
Continuous function in this interval.

(b) Infinite type (at least one of the twolimit are infinity)
x f (1 )   
(i) f(x) = at x = 1
1 x f (1 )   

  
f 0
2 
 

(ii) f(x) = 2tanx at x =
2   
f  
2 
 

1 f (0  )  
(iii) f(x) = 2 at x = 0
x f (0  )  

(c) Oscillatory (limits oscillate between two finite quantities)


1
f ( x )  sin
x 
(i) or  at x = 0 oscillates between – 1 & 1
1 
f ( x )  cos
x
 1 
(ii) f(x) = 1  sin( ln | x | )  at x = 0 oscillates between 0 & 1.
 3 

BANSAL CLASSES Private Ltd. ‘Bansal Tower’, A-10, Road No.-1, I.P.I.A., Kota-05 Page # 12
LIMIT, CONTINUITY AND DIFFERENTIABILITY OF FUNCTION

6. CONTINUITY OF FUNCTIONS DEFINED BY SOME FUNCTIONAL


RULE :

If f(x+y) = f(x) . f(y) for all x & y and f(x) = 1 + g(x). G(x) where Lim g(x)= 0 & Lim G(x) exist.
x 0 x 0

7. THEOREMS ON CONTINUITY :

T–1 : Sum, difference, product and quotient of two continuous functions is always a continuous function.
f (x )
However h (x) = is continuous at x = a only if g (a)  0.
g( x)

8. FOLLOWING IMPORTANT NOTES SHOULD BE REMEMBERED :

(a) If f (x) is continuous and g (x) is discontinuous then f (x) + g (x) is a discontinuous function.

(b) If f(x) is continuous & g(x) is discontinuous at x = a then the product function (x) = f(x) . g(x) is not
necessarily discontinuous at x = a. e.g.
sin 1 x0
(i) f(x) = x & g(x) =  0 x x 0.

 1
x sin , x0
Then f(x) · g(x) =  x is continuous at x = 0.
0, x0

 2x  1 
(ii) f (x) = cos   is continuous at x = 1 and g (x) = [x] and [ · ] denote the greatest integer
 2 
functions is discontinuous at x = 1, but f (x) · g (x) is continuous at x = 1.
 x  1  
Lim cos   · [ x ]  cos · 1  0
x  1  2  2

 x  1  
Lim cos    · [ x ]  cos  · 0   0, f (1)  0  continuous at x = 1.
x 1
 2  2

(c) If f(x) and g(x) both are discontinuous at x = a then the product function (x) = f (x) . g(x) is not
necessarily discontinuous at x = a. e.g.
1 x0
f(x) =  g(x) = 
 1 x  0
 f(x) g(x) = 1  x  R which is continuous function.

BANSAL CLASSES Private Ltd. ‘Bansal Tower’, A-10, Road No.-1, I.P.I.A., Kota-05 Page # 13
LIMIT, CONTINUITY AND DIFFERENTIABILITY OF FUNCTION

T–2: Intermediate value theorem :

If f is continuous on [a, b] and f (a)  f (b) then for any value c  f (a ), f (b)  , there is at least one
number x0 in (a, b) for which f (x0) = c

y
y
f(b) f(b)
c
c
f(a)

a x
0 x=a x0 x=b x1 0 x2 x3 b

f(a)
Figure-1

y y
f(b)
c
f(a)
f(b)

a x
0 a b 0 b

Figure-2 f(a) Figure-3

NOTE:
(i) Continuity through the interval [a, b] is essential for the validity of this theorem.
(ii) in figure-3,f (a)and f (b) are ofopposite signbut f(x) has noroot in(a, b)as f is continuous.

T-3: Extreme Value Theorem :

If f is continuous on [a, b] then f takes on, a least value m and a greatest value M on this interval.

M M

m
m
a c d b a b
Minimum value 'm' occurs Minimum value 'm' occurs at the
at x = c and maximum value end point x = a and the maximum
M occurs at x = d. c, d  (a, b) value M occurs inside the interval

BANSAL CLASSES Private Ltd. ‘Bansal Tower’, A-10, Road No.-1, I.P.I.A., Kota-05 Page # 14
LIMIT, CONTINUITY AND DIFFERENTIABILITY OF FUNCTION

Note :
To see that continuity is necessary for the extreme value theorem to be true refer the graph shown.

There is a discontinuity at x = c in the interval. The


function has a minimum value at the left end point x = a
and f
has no maximum value.

9. PROPERTIES OF FUNCTION CONTINUOUS IN [a, b] :

(i) If a function f is continuous on a closed interval [a, b] then it is bounded.


(ii) A continuous function whose domain is some closed interval must have its range also in closed interval.
(iii) If f is continuous and onto on [a, b] then f –1(from the range of f ) is also continuous.
(iv) If f (a) and f (b) possess opposite signs then  at least one solution of the equation f (x) = 0 in the open
interval (a, b) provided f is continous in [a, b].

10. CONTINUITY OF SPECIAL TYPES OF FUNCTIONS :


10.1 Continuity of functions in which greatest integer function is involved:
f (x) = [x] is discontinuous when x is an integer.
Similarly, f (x)= [g (x)] is discontinuous when g(x) is an integer, but this is true onlywhen g(x) is monotonic
(g(x) is strictlyincreasing or strictly decreasing).
For example, f (x) = [ x ] is discountinuous when x is an integer, as x is strictly increasing
(monotonic function).

3
2
1
1 4 9 16

f (x) = [x2], x  0 is discountinuous when x2 is an integer, as x2 is strictly increasing for x  0.

1 2 3 25

BANSAL CLASSES Private Ltd. ‘Bansal Tower’, A-10, Road No.-1, I.P.I.A., Kota-05 Page # 15
LIMIT, CONTINUITY AND DIFFERENTIABILITY OF FUNCTION

Now consider, f (x) = [sin x], x  [0, 2]. g(x) = sin x is not monotonic in [0, 2].

For this type of function, points of discontinuity can determined easily by graphical methods. We can
3 3
note that at x  , sin x takes integral value –1, but at x = , f (x) = [sin x] is continuous.
2 2
y

1
x
O 2
–1

10.2 Continuity of functions is which signum function is involved :


We know that f (x) = sgn (x) is discontinuous at x = 0.
In general, f (x) = sgn (g (x)) is discontinuous at x = a if g(a) = 0.

10.3 Continuity of functions involving limit Lim an :


n

0, 0  a  1

We know that lim a  1, a  1
n
n 
, a  1

10.4 Continuity of functions in which f (x) is defined differently for rational


and irrational values of x :
Example

(1) f (x) =  0x ifif xxQQ (2) f (x) =  x x ifif xxQQ


continuous only at x = 0 continuous only at x = 0

(3) f (x) =  x if x  Q
1  x if x  Q (4) f (x) =  x 2 if x  Q
1 if x  Q
continuous only at x = 1/2 continuous only at x = 1 or – 1

11. CONTINUITY OF COMPOSITE FUNCTIONS :


If f is continuous at x = a & g is continuous at x = f (a) then the composite, g[f(x)], is continuous
x sin x
at x = a. e.g. f (x) = & g (x) = | x | are continuous at x = 0, hence the composite
x2  2

x sin x
(gof) (x) = will also be continuous at x = 0.
x2  2

BANSAL CLASSES Private Ltd. ‘Bansal Tower’, A-10, Road No.-1, I.P.I.A., Kota-05 Page # 16
LIMIT, CONTINUITY AND DIFFERENTIABILITY OF FUNCTION

DIFFERENTIABILITY

1.0 DIFFERENTIABILITY / DERIVABILITY:


(Two fold meaning of derivability)

Geometrical meaning of derivative Physical meaning of derivative


Slope of the tangent drawn to the curve (functions which are differentiable)
at x = a if it exists Instantaneous rate ofchange of function

Note : "Tangent at a point 'A' is the limiting case of secant throughA."

1.1 RIGHT AND LEFT HAND DERIVATIVES :

1.2 EXISTENCE OF DERIVATIVE :


f (a  h )f (a )
(I) Right hand & Left hand Derivatives ; By definition : f (a) = Lim if it exists
h 0 h
f (a  h )f (a )
(i) The right hand derivative of f at x = a denoted by f (a+) is defined by :f ' (a+) = Lim ,
h 0 h
provided the limit exists & is finite.
when h  0, the point B moving along the curve tends toA, i.e., B Athen the chordAB approaches
the tangent line AT at the point A and then   
f (a  h )  f (a )
 f ' (a+) = lim  lim tan   tan 
h 0 h h 0

f (a  h )f (a )
(ii) The left hand derivative of f at x = a denoted by f  (a–) is defined by : f ' (a–) = Lim ,
h 0  h
Provided the limit exists & is finite.
When h  0, the point C moving along the curve tends toA, i.e., C Athen the chord CAapproaches
the tangent lineAT at the point Athen

BANSAL CLASSES Private Ltd. ‘Bansal Tower’, A-10, Road No.-1, I.P.I.A., Kota-05 Page # 17
LIMIT, CONTINUITY AND DIFFERENTIABILITY OF FUNCTION

f ( a  h )  f (a )
f ' (a–) = lim
h 0 h
'f ' is said to be derivable at x = a if f ' (a+) = f ' (a– ) = a finite quantity.
This geometrically means that a unique tangent with finite slope can be drawn at x = a as shown in the
figure.

(II) Theorem :
If a function 'f' is derivable at x = a then 'f' is continuous at x =a.
Proof :
Let 'f ' be derivable at x = a. Hence
f (a  h )  f ( a )
f (a) = Lim exists .
h 0 h
f ( a  h )  f (a )
Also f (a  h )  f (a )  ·h (h  0)
h
f (a  h )  f (a )
Therefore : Lim [(f (a + h) – f (a)] = Lim .h  f ' (a ).0  0
h 0 h 0 h
Therefore Lim [(f (a + h) – f (a)]= 0  Lim f (a + h) = f (a)  f is continuous at x.
h 0 h 0

Note: If f(x) is derivable for every point of its domain of definition, then it is continuous in that domain .
The Converse of the above result is not true :
For a function 'f ' :
Differentiability Continuity ; Continuity  derivability ;
Non derivability 
 discontinuous But discontinuity Non derivability

Note
(a) Let f (a+) = p & f _(a) = q where p & q are finite then :
(i) p = q  f is derivable at x = a  f is continuous at x = a .
(ii) p  q  f is not derivable at x = a.
It is very important to note that f may be still continuous at x = a.

(b) If a function f is not differentiable but is continuous at x = a it geometrically implies a sharp corner
at x = a.

(III) How can a function fail to be differentiable :

The function f (x) is said to non-differentiable at x = a if


(a) Both left and right hand derivative exists but are not equal y

The function y = |x| is not differentiable at 0 as its graph change


direction abrupty when x = 0. In general, if the graph of a function
has a 'corner' or 'kink' in it, then the graph of f has no tangent at this
point and f is not differentiable there. (To compute f ' (a), we find that x
O a
the left and right limits are different.) A corner

BANSAL CLASSES Private Ltd. ‘Bansal Tower’, A-10, Road No.-1, I.P.I.A., Kota-05 Page # 18
LIMIT, CONTINUITY AND DIFFERENTIABILITY OF FUNCTION

y
(b) Function is discontinuous at x = a

If 'f ' is not continuous at 'a' then f is not differentiable at 'a'. So at any
discontinuity (for instance, a jump of discontinuity) 'f ; fails to be a
x
O
differentiable. A discontinuity

(c) Either or both left and right hand derivative are not finite. y

A third possibility is that the curve has a vertical tangent line when
x = a, that is 'f ' is continuous at a and lim | f ' ( x ) | = .
xa
a
x
O
This means that the tangent lines becomes steeper and steeper A vertical tangent
as x  a.

(IV) Derivability Over An Interval :

f (x) is said to be derivable over an open interval (a, b) if it is derivable at each & every point of the
interval f(x) is said to be derivable over the closed interval [a, b] if :

(i) for the points a and b, f (a+) & f (b ) exist &

(ii) for any point c such that a < c < b, f (c+) & f(c ) exist & are equal.

Note : Consider the graph of a differentiable function


(1) If f (x) is derivable at x = x0 then | f(x) | must be
derivable at x = x0 provided f (x0)  0. However
if f (x0) = 0 then | f (x) | may or may not be
derivable at x = 0

e.g. f (x) = x3 is derivable at x = 0 and | f (x) | is also derivable at x = 0.

f (x) = x – 1 is derivable at x = 1 but | f (x) | is not derivable at x = 1


If f ' (x0) = 0 and f (x0) = 0 then | f (x) | will also be derivable at x = x0 and if f ' (x0) is a non zero
finite
then | f (x) | is non derivable at x = x0.

In the figure f(x) = 0 at A, B, C and f(x) is derivable at A, B and C but | f(x) | is non derivable at
x =B and C but derivable at x = A.
Consider f(x) = x3 at x = 0 when f '(0) = 0 and f(0) = 0 but from | f(x) | = |x3| is derivable but if
f (x) = x, where f ' (x)  0,
hence | f (x) | is not derivable at x = 0

BANSAL CLASSES Private Ltd. ‘Bansal Tower’, A-10, Road No.-1, I.P.I.A., Kota-05 Page # 19
LIMIT, CONTINUITY AND DIFFERENTIABILITY OF FUNCTION

(V) Theorem :

If f (x) and g (x) both are derivable at x = a, then

(i) f (x) ± g (x) will be differentiable at x = a.

(ii) f (x) · g (x) will be differentiable at x = a.


f (x )
(iii) will be differentiable at x = a if g (a)  0.
g( x )

Note that :
f (x )
(1) If f (x) and g (x) are both derivable at x = a, f (x) ± g (x); g (x) · f (x) and will also be derivable at
g( x )
x = a. (only if g (a)  0)

(2) If f (x) is derivable at x = a and g (x) is not derivable at x = a then the f (x) + g (x) or f (x) – g (x) will not
be derivable at x = a.
e.g. f (x) = cos | x | is derivable at x = 0 and g (x) = | x | is not derivable at x = 0,
then cos | x | + | x | or cos | x | – | x | will not be derivable at x = 0.
However nothing can be said about the product function in this case.
f (x) = x is derivable at x = 0
g (x) = | x | is not derivable at x = 0

 x 2 if x  0
x|x|=  2
 x if x  0

(3) If both f (x) and g (x) are non derivable then nothing definite can be said about the
sum / difference / product function.
e.g. f (x) = sin | x | is not derivable at x = 0
g (x) = | x | is not derivable at x = 0
then the function
F (x) = sin | x | – | x | is derivable at x = 0
G (x) = sin | x | + | x | is not derivable at x = 0.

(4) If f (x) is derivable at x =a and f (a) = 0 and g (x) is continuous at x = a then the product function
F (x) = f (x) · g (x) will be derivable at x = a
f (a  h )g (a  h )  0
F ' (a+) = Lim = f ' (a) · g (a)
h 0 h
f (a  h ) g (a  h )  0
F ' (a–) = Lim = f ' (a) · g (a)
h 0 h

BANSAL CLASSES Private Ltd. ‘Bansal Tower’, A-10, Road No.-1, I.P.I.A., Kota-05 Page # 20
LIMIT, CONTINUITY AND DIFFERENTIABILITY OF FUNCTION

(5) Derivative of a continuous function need not be a continuous function


1
2
 x sin x if x  0 f ' (0  )  0
f (x) =  ; 
0 if x  0 f ' (0 )  0

1 1  1 
 sin x · 2x  x cos x  x 2  if x  0
2

f ' (x) = 
 0 if x  0
f ' (x) is not continuous at x = 0.

(VI) Determination of function which are differentiable and satisfying


the given functional rule :

BASIC STEPS :
f (x  h)  f (x)
(i) Write down the expression for f ' (x) as f ' (x) =
h

(ii) Manipulate f (x + h) – f (x) in such a way that the given functional rule is applicable. Now apply the
functional rule and simplify the RHS to get f ' (x) as a function of x along with constants if any.

(iii) Integrate f ' (x) get f (x) as a function of x and a constant of integration. In some cases a Differential
Equation in formed which can be solved to get f (x).

(iv) Apply the boundary value conditions to determine the value of this constant.

BANSAL CLASSES Private Ltd. ‘Bansal Tower’, A-10, Road No.-1, I.P.I.A., Kota-05 Page # 21
LIMIT, CONTINUITY AND DIFFERENTIABILITY OF FUNCTION

EXERCISE-1 [SPECIAL DPP]

SPECIAL DPP-1 [LIMIT]

 1 if x  0

Q.1 The signum (or sign) function, denoted by sgn, is defined by sgn x = 0 if x  0 .
1 if x  0
Compute the following.
(i) Lim sgn x (ii) Lim sgn x (iii) Lim sgn x (iv) Lim | sgn x |
x 0 x 0 x0 x0

Q.2 For the function g whose graph is given, state the value of each quantity, if it exists.
(a) Lim g ( t ) (b) Lim g( t ) y
t 0 t 0
4
(c) Lim g( t ) (d) Lim g( t )
t0 t 2
2

(e) Lim g( t ) (f) Lim g( t ) 0


t 2 t2 2 4 t
–1
(g) g(2) (h) Lim g( t )
t4 –2

2x 2  3x
Q.3 Lim .
x
3 | 2x  3 |
2

f (x ) f (x )
Q.4(a) If Lim 2
= 1, find (i) Lim f ( x ) ; (ii) Lim
x 2 x x 2 x  2 x

f (x) f (x)
(b) If Lim 2
= 1, find (i) Lim f ( x ) ; (ii) Lim
x 0 x x0 x0 x

3x 2  ax  a  3
Q.5 The value of a for which Lim exists.Also find the value of the limit.
x 2 x2  x  2
P( x )
Q.6 Let P and Q be polynomials. Find Lim if the degree of P is
x   Q( x )

(a) less than the degree of Q and (b) greater than the degree of Q.

Q.7 Determine whether the statement is true or false.


 2x 8  2x 
(a) Lim     Lim  Lim .
x 4  x  4 x4 x  4 x4 x  4 x4

x 2  6 x  7 Lim x 2  6x  7  
 x 1
 
(b) Lim 2 .
x 1 x  5x  6
Lim x 2  5x  6
x 1

BANSAL CLASSES Private Ltd. ‘Bansal Tower’, A-10, Road No.-1, I.P.I.A., Kota-05 Page # 22
LIMIT, CONTINUITY AND DIFFERENTIABILITY OF FUNCTION

x 3 Lim ( x  3)
x 1

 
(c) Lim .
x 1 x 2  2 x  4 Lim x 2  2 x  4
x 1

f (x )
(d) If Lim f ( x )  2 and Lim g( x )  0 , then Lim does not exist.
x 5 x 5 x5 g ( x )

f (x )
(e) If Lim f ( x )  0 and Lim g( x )  2 , then Lim does not exist.
x 5 x 5 x5 g ( x )

3
1 x  3 1 x ( x  1)10  ( x  2)10  .......  ( x  100)10
Q.8 Lim . Q.9 Lim .
x0 x x x10  1010

7  x3  3  x2
 x  a  x  b   x  .
3
Q.10 Lim . Q.11 Lim
x 1 x 1 x

ax
Q12 Lim a  0 .
x    a x 1
Y

Q.13 Refer to the graph of y = f (x)


and g(x) = (x – 2)2, x < 2 4
= 7 – x, x  2 3
then which of the following limits are non existent. f(x) 2

(a) Lim f (g( x )) (b) Lim g(f ( x )) (c) Lim g(f ( x ))


x2 x0 x5 X
O 1 2 3 5

  is equal to
n n
Q.14 For n N, let an =  2k and b n   (2k  1) . Then nLim

a n  bn
k 1 k 1

1
(A) 1 (B) (C) 0 (D) 2
2

Q.15 If Lim [ f ( x )  g ( x )]  2 and Lim [ f ( x )  g ( x )]  1 , then Lim f ( x ) g ( x )


x a x a x a

3 3 4
(A) need not exist (B) exist and is (C) exists and is (D) exists and is
4 4 3

Q.16 Limit

cot 1 x  1  x is equal to 
x   x
1  2 x  1  
sec   
 x  1  
(A) 1 (B) 0 (C) /2 (D) non existent

BANSAL CLASSES Private Ltd. ‘Bansal Tower’, A-10, Road No.-1, I.P.I.A., Kota-05 Page # 23
LIMIT, CONTINUITY AND DIFFERENTIABILITY OF FUNCTION

x2  x  2
Q.17 The value of Lim is equal to
x 2
x2  4
1 1
(A) 1 (B) 2 (C) (D)
2 4

n
n!
Q.18 Let Un =
(n  2)!
where n  N. If Sn =  Un then Lim Sn equals
n 
n 1

1
(A) 2 (B) 1 (C) (D) non existent
2

Q.19 The graph of functions f and g are shown below.

y y

2 2
y=f(x) y=g(x) 4/3
1
1
–2 –1 1 2 x –2 –1 1 2 x
O O

–1 –1

–2 –2

[Note : [k] denotes the greatest integer less than or equal to k.]
Consider the following statements

I. Lim  f ( x )  g( x )  exist and is equal to 2.


x 1

II. Lim  f ( x )  g( x )  exist and is equal to 1.


x 2

III. Lim  f g ( x )   exist and is equal to 1.


x0

IV. Lim  g f ( x )   exist and is equal to – 1.


x2

Which of the statements I, II, III and IV given above are correct?
(A) I, II and III (B) I, II, III and IV (C) I, II and IV (D) II, III and IV

x 1
Q.20 Let f(x) = . Then
2 x  7x  5
2

1 1
(A) Limit
x1
f(x) =  (B) Limit

f(x) =  (C) Limit
x   f(x) = 0 (D) Limit
x  5/ 2
does not exist
3 x 0 5

BANSAL CLASSES Private Ltd. ‘Bansal Tower’, A-10, Road No.-1, I.P.I.A., Kota-05 Page # 24
LIMIT, CONTINUITY AND DIFFERENTIABILITY OF FUNCTION

Q.21 Which of the following limit do not exist?


x3  x  2  1 1 
(A) Lim 3 (B) Lim   2 
x 1 x  x2  x 1 x  2 x ( x  2) 2
x  3x  2 

 x2 x4   x3 x 2 
(C) Lim    (D) Lim  2 
x  1 x  5x  4 3( x  3x  2)  2 x  1 
2 2
 x   2x  1

Q.22 Which of the followinglimits vanish?


sin x arc tan x
(A) Lim (B) Lim
x x x  x
x  sin x arc sin x
(C) Lim (D) Lim
x x  cos x x 1 x
tan
2

SPECIAL DPP-2 [LIMIT]

x2 x sin x x sin x
Q.1(a) If the inequalities 1 –  < 1 hold for all values of x close to zero. Find Lim .
6 2  2 cos x x 0 2  2 cos x

4x  1 4x 2  3x
(b) Find Lim f ( x ) if  f (x )  for all x > 5.
x  x x2
n
 k 
(c) Find the value of Lim   2 .
k 1  n  k 
n 

n
r2
(d) Find the value of Lim
n
 n3  n 2  r
.
r 1

  x
sin  x   1  sin
Q.2 Lim  6
. Q.3 Lim 2 .
 3 x x x x
x
6  cos x cos  cos  sin 
2 2 4 4

  arc cos x
Q.4 Lim 2 x tan x   sec x  . Q.5 Lim .
x 2 x  1 x 1

n  1  a
Q.6 Let Pn =  1  k 1 
C2 
. If Lim Pn can be expressed as lowest rational in the form , find the
n  b

k 2

value of (a + b).

BANSAL CLASSES Private Ltd. ‘Bansal Tower’, A-10, Road No.-1, I.P.I.A., Kota-05 Page # 25
LIMIT, CONTINUITY AND DIFFERENTIABILITY OF FUNCTION

sin x x  n, n  0,  1,  2,  3....... x 2  1, x  0, 2


  4, x  0
Q.7 If f (x) =  and g (x) = 
 2 otherwise  5, x  2

then Lim g f(x) is equal to


x0
(A) 1 (B) 4 (C) 5 (D) non existent

cos 2  cos 2 x
Q.8 Lim =
x 1 x2  | x |
(A) 2 cos 2 (B)  2 cos 2 (C) 2 sin 2 (D)  2 sin 2

Q.9 
If Lim ln ( x 2  5x )  2 ln (cx  1) = – 2 then
x 

1
(A) c = e–1 (B) c = e–2 (C) c = e (D) c = e2

x  6  sin( x  3)  3
Q.10 The value of Lim is equal to
x 3 ( x  3) cos( x  3)
1 3 5
(A) (B) 5 (C) (D)
5 4 6
1  cos ax sin x
Q.11 For a  R+ such that Lim  Lim , then the value of a is
x 0 x 2 x    x

2
(A) 1 (B) 2 (C) 2 (D)
2

 1  n
1 
2
Q.12 The value of the limit is
n 2

1 1 1
(A) 1 (B) (C) (D)
4 3 2

Q.13 If Lim f ( x )  p and Lim f ( x )  q then the correct statement(s) is/are true?
x 0 x 0 

(A) Lim f ( x  x )  q (B) Lim f ( x  x )  p


3 3
x 0 x 0

(C) Lim f ( x  x )  q (D) Lim f ( x  x )  p


2 4 2 4
x 0 x 0

cos 1 cos x 
Q.14 Let f(x) = , then which of the following is(are) correct ?
x
[Note : [x] denotes greatest integer less than or equal to x.]
(A) Lim f ( x )  1 (B) Lim f ( x )   2 (C) Lim f ( x )  0 (D) Lim f ( x )  1
x0 x 0 x 0 x0 

BANSAL CLASSES Private Ltd. ‘Bansal Tower’, A-10, Road No.-1, I.P.I.A., Kota-05 Page # 26
LIMIT, CONTINUITY AND DIFFERENTIABILITY OF FUNCTION

Q.15 Graph of a function y = f (x) is shown in the adjacent figure.


Four limits l1, l2, l3 and l4 are given as : f (x)
l1 = Lim [f ( x )  2] 4
x 0 


l2 = Lim f ( x 2 )  f ( x )
x 0
 3

2

l3 = Lim sin 1 sin f [ x ]  f (0)
x 0
 x
–2 0 2
  
l4 = Lim  sin 1 sin f ( x ) 2  tan 1 tan f ( x )  
x 0   
then find the value of (l1 + l2 + l3 + l4).
[Note : [k] denotes greatest integer function less than or equal to k.]

SPECIAL DPP-3 [LIMIT]

Evaluate Q.1 to Q.4:


 x x x   x 1 x 
Q.1 Lim  cos · cos ...... cos n  . Q.2 Lim x  arc tan  arc tan .
n  2 4 2  x  x2 x2

 x 1   ex  cos x
Q.3 Lim x  arc tan  . Q.4 Lim x .
x  x2 4 x 0 e  cos  x

1  x , 0  x  1
Q.5 Consider the function f (x) =  x  2, 1  x  2 . Find Lim f f ( x )  and Lim f f ( x )  .
4  x , 2  x  4 x1 x2

 sinx   2 sin 2 x  10 sin 10 x 


Q.6 Let f (x) =      ............    (where [y] is the largest integer y).
 x   x   x 
The value of Lim f ( x ) equals
x0
(A) 55 (B) 164 (C) 165 (D) 375

5x · 1  2 x  cos x
Q.7 Limit equals
x 0 x
(A) ln 5 (B) ln 5e (C) 2 + ln 5 (D) none

2 2  (cos x  sin x )3 
Q.8 The limiting value of the function f(x) = when x  is
1  sin 2 x 4
1 3
(A) 2 (B) (C) 3 2 (D)
2 2

BANSAL CLASSES Private Ltd. ‘Bansal Tower’, A-10, Road No.-1, I.P.I.A., Kota-05 Page # 27
LIMIT, CONTINUITY AND DIFFERENTIABILITY OF FUNCTION

sec x   tan x   1
Q.9 The value of Lim is equal to
x 2 x2
(A ) sec2 · ln sec  + tan2 · ln tan  (B) sec2 · ln tan  + tan2 · ln sec 
(C) sec2 · ln tan  – tan2 · ln sec  (D) sec2 · ln sec  – tan2 · ln tan 

a sin x  1 x 1
Q.10 If Lim = Lim (where a > 0), then a is equal to
x 0 1  x  cos x x 1 ln ( x )

1 1
(A) (B) (C) e (D) 1
2 e

tan 2x
 Lim
1  cos 2 x 2
Q.11 For  > 0, let Lim , then  is equal to
x x x0 x4
1 1
 
(A) 2 4 (B) 2 2 (C) 2–1 (D) 1

1
(4x  1) 3  a  bx 1
Q.12 If Lim exist and is equal to , then
x 0 x 3
(A) a + b = 0 (B) a – b = – 2 (C) a – b = 2 (D) a2 + b2 = 4

sin 2 4x  2 cos2 x  2 cos x


Q.13 Lim .
x 0 cos 2 x  cos3 x

 
Q.14 If Lim x   x 2  x 4  1  2 x  exists and has value non-zero finite real number L,
x   

then find the value of .
L2

SPECIAL DPP-4 [LIMIT]

x
 x 2  2x  1 
Q.1 Lim  2  =
x   x  4 x  2 
 
(A) 1 (B) 2 (C) e2 (D) e
1
Q.2 Lim (lnx ) e
x is
x e 

1
(A) ee (B) ee (C) e–2 (D) e–1

BANSAL CLASSES Private Ltd. ‘Bansal Tower’, A-10, Road No.-1, I.P.I.A., Kota-05 Page # 28
LIMIT, CONTINUITY AND DIFFERENTIABILITY OF FUNCTION

6 x 1
 3x 2  2x  1 3x  2
Q.3 L et f(x) =  2  then Lim 1  f ( x ) 2 has the value equal to
 x x2  x 
(A) 10 (B) 100 (C) 1000 (D) none

Q.4 Let f(x, y) = sin (2x – y) cos y + cos (2x – y) sin y  x, y  R.


1


The value of Lim 1  tan x  x
x0
 2
x f ( x,y) is equal to

1 1 1 1
(A) e 3 (B) e 3 (C) e 6 (D) e 6

nx n 1  ( n  1) x n  1
Q.5 Lim where n = 100 is equal to
x 1 (e x  e) sin x
5050 100 5050 4950
(A) (B) (C) – (D) –
e e e e

tan 1 nx 
2
Q.6 Which one of the following best represents the graph of the function f(x) = Lim
n  

(A) (B) (C) (D)

2
Q.7 Limit  1  log2 x cos x
x0  cos
2 
(A) is equal to 4 (B) is equal to 9 (C) is equal to 289 (D) is non existent

sin x cos x n
Q.8 Let a = min [x2 + 2x + 3, x  R] and b = Lim
x 0 e x  ex
. Then the value of  a r b n r is
r 0

2 n 1  1 2 n 1  1 2n  1 4 n 1  1
(A) (B) (C) (D)
3· 2n 3 · 2n 3· 2n 3 · 2n

Q.9 The natural number n, for which Lim


27 x

 9x  3x  1 cos x  e xis a finite non zero number, is
x 0 x 2 n 1
equal to
(A) 1 (B) 2 (C) 3 (D) 4

BANSAL CLASSES Private Ltd. ‘Bansal Tower’, A-10, Road No.-1, I.P.I.A., Kota-05 Page # 29
LIMIT, CONTINUITY AND DIFFERENTIABILITY OF FUNCTION

f (x)  2 2
Q.10 If f (x) is a quadratic expression such that f (–2) = f (2) = 0 and f (1) = 6 then Lim
x 0 ln (cos x )
is equal to
1 1
(A) –4 (B) 4 (C) (D) –
2 2

 1  n
 n  
 ( n )   1  a
Q.11 Let a  n
ln
and b   n   . The value of Lim   is equal to
n b
 ln ( n ) 
1
(A) 0 (B) 1 (C) e (D)
e

1
Q.12 Let f (x) = x sin , then the correct statement is(are)
x
(A) Lim f (x )  0 (B) Lim f (x)  1 (C) Lim f (x)  1 (D) f (0) = 0
x 0 x  x 

Q.13 Column-I Column-II


 x2 x 3 
(A) Lim x 2  3  equals (P) 0
x 
 x x 

x 2 (1  sin 2 x ) 1
(B) Lim (Q)
x  ( x  sin x ) 2
2

3 2  1
Let a, b  R. If Lim  4x  x  ax  b  =
3
(C) (R) 1
x    3
then the sum (a + b) equals (S) 2
n(1  x 2  x 4 )
(D) Lim . (T) does not exist
x 0 (e x  1)x
x  2 sin x
(E) Lim
x 0
x 2  2 sin x  1  sin 2 x  x  1

Q.14 If the angles A, B, C (in that order) of triangle ABC are in arithmetic progression,
3  4 sin A sin C
and L = Lim , then find the value of (100L2).
A C |AC|

BANSAL CLASSES Private Ltd. ‘Bansal Tower’, A-10, Road No.-1, I.P.I.A., Kota-05 Page # 30
LIMIT, CONTINUITY AND DIFFERENTIABILITY OF FUNCTION

SPECIAL DPP-5 [LIMIT]

x 2n  1
Q.1 The function f (x) = Lim 2 n is identical with the function
n  x 1
(A) g (x) = sgn(x – 1) (B) h (x) = sgn (tan–1x)
(C) u (x) = sgn( | x | – 1) (D) v (x) = sgn (cot–1x)
A
Q.2 The figure shows an isosceles triangleABC with B = C.
The bisector of angle B intersects the side AC at the point P.
Suppose that BC remains fixed but the altitude AM approaches 0, so
that A M (mid-point of BC). Limiting value of BP, is P

a a
(A) (B)
3 2 B C
M
2a 3a
(C) (D)
3 4
where a is fixed side BC.

x
 ( x  1) x 
Q.3 Lim  x  equals

x 
 x ·e 
1 1
(A) e (B) (C) e (D)
e e

Q.4 Lim  n 2  n  1   n 2  n  1   (n  I) where [ ] denotes the greatest integer function is


n     

1 2 1
(A) 0 (B) (C) (D)
2 3 4

12 n  2 2 (n1)32 (n2).....n 2.1


Q.5 Lim is equal to
n  13 23 33 ......n 3
1 2 1 1
(A) (B) (C) (D)
3 3 2 6

Paragraph for question nos. 6 to 8


 1  x2 1   
 and g(x) = cosec–1  1  x
2
Consider, f(x) = tan–1   , x 0
 x   x 
   

Q.6 The number of solution(s) of the equation


x2 = | f(x) – g(x) | is
(A) 0 (B) 1 (C) 2 (D) 3

BANSAL CLASSES Private Ltd. ‘Bansal Tower’, A-10, Road No.-1, I.P.I.A., Kota-05 Page # 31
LIMIT, CONTINUITY AND DIFFERENTIABILITY OF FUNCTION


Q.7 If f(x) + g(x) = then x equals
8
1
(A) 2 – 3 (B) 2 + 3 (C) 3 (D)
3

 6f ( x )  g ( x )  
Q.8 The value of Lim   equals
x 0  x 
[Note: [k] denotes greatest integer function less than or equal to k.]
(A) 9 (B) 8 (C) 6 (D) 5

 x
 tan x ,  x  0
 x  4,   x 1
  2
Q.9 Let f (x) = 2, x0 and g (x) = x  5x  11, 1 x  2 ,
 x  3, 2x

 ln (1  2 x )
 , 0x
 x

then which of the following is(are) correct ?


(A) Lim g f ( x ) = 5 (B) Lim gf ( x ) = 7
x0 x0

(C) Lim g f ( x ) = 5 (D) Lim gf ( x ) = – 1


x  0 x  0

Q.10 The graph of y = f(x) is as shown.


Then which of the following hold(s) good?
[Note : [k] and {k} denote largest integer less than or equal to k and fractional part of k respectively.]
y
(A) Lim f  x  sin x    4
x 0  7

6
(B) Lim f  x  tan x    2
x 0 5

4
  x 
(C) Lim f      7
x 0    tan x   3

2
(D) Lim f cos x  5
 1
x
2
x
–1 O 1

BANSAL CLASSES Private Ltd. ‘Bansal Tower’, A-10, Road No.-1, I.P.I.A., Kota-05 Page # 32
LIMIT, CONTINUITY AND DIFFERENTIABILITY OF FUNCTION

Q.11 Column-I Column-II

(A) Lim  x  x  x  x  equals (P) –1


x   

tan 3 x  tan x 3
(B) The value of Lim equals (Q) 0
x 0 x5

(C)
x 0 
 
Lim ln sin 3 x  ln ( x 4  ex 3 ) equals (R) 1

k2  k
n p
(D) If Lim
n
 n 3
 k
can be expressed as rational
q
in the lowest form, (S) 2
k 1

then the value of (p + q), is (T) 4

Q.12 For a certain value of c, Lim [(x5 + 7x4 + 2)C - x] is finite and non zero and has the value equal to l.
x 
Find the value of (l – 2c).

1  a cos 2x  b cos 4x
Q.13 If Lim exists for all x  R and has the value equal to c.
x 0 x4
Find the value of 3(a + b + c).

  1 
1 x
m
Q.14 Let Lim x ln  e1    equal where m and n are relatively prime positive integer..
x    x  n
Find (m + n).

SPECIAL DPP-6 [CONTINUITY]

Q.1 The following functions are not defined at x = 0. Find f (0) so that f is continuous at x = 0.
(3x  1) 2 2  4 x 2  16
(a) f (x) = (b) f(x) =
sin x . ln (1  x ) cos 2x  1

1 2 a  xa2
(c) f(x) =  2x (x  0) (a > 0 and b  0)
 
(d) f(x) =
x e 1 sin bx

cos x  sin(x 2)
if x  1
 ( x  1)(3x 2  2 x  1)
Q.2 If f (x) =  is continuous at x = 1, then find the value of k .

k if x  1

BANSAL CLASSES Private Ltd. ‘Bansal Tower’, A-10, Road No.-1, I.P.I.A., Kota-05 Page # 33
LIMIT, CONTINUITY AND DIFFERENTIABILITY OF FUNCTION

1
Q.3 For the function f (x) = 1
, x  2 which of the following holds?
( x 2)
x2
(A) f (2) = 1/2 and f is continuous at x =2 (B) f (2)  0, 1/2 and f is continuous at x = 2
(C) f can not be continuous at x = 2 (D) f (2) = 0 and f is continuous at x = 2.
4  x2
Q.4 The function f (x) = , is
4x  x 3
(A) discontinuous at only one point in its domain.
(B) discontinuous at two points in its domain.
(C) discontinuous at three points in its domain.
(D) continuous everywhere in its domain.

x 2  bx  25
Q.5 If f (x) = 2 for x  5 and f is continuous at x = 5, then f (5) has the value equal to
x  7 x  10
(A) 0 (B) 5 (C) 10 (D) 25
x  e x  cos 2x
Q.6 If f(x) = , x  0 is continuous at x = 0, then
x2
5
(A) f (0) = (B) [f(0)] = – 2 (C) {f(0)} = –0.5 (D) [f(0)] . {f(0)} = –1.5
2
where [x] and {x} denotes greatest integer and fractional part function

(2 x  1)
Q.7 The function f(x) = [x] . cos , where [•] denotes the greatest integer function, is discontinuous
2
at
(A) all x (B) all integer points
(C) no x (D) x which is not an integer

loge (1  3 f (x))
Q.8 y = f (x) is a continuous function such that its graph passes through (a, 0). Then Lim
x a is
2 f (x)
3 2
(A) 1 (B) 0 (C) (D)
2 3

Q.9 The function f(x) = [x]2  [x2] (where [y] is the greatest integer less than or equal to y), is discontinuous
at
(A) all integers (B) all integers except 0 & 1
(C) all integers except 0 (D) all integers except 1

Q.10 f is a continuous function on the real line. Given that


x2 + (f(x)  2) x  3 . f(x) + 2 3  3 = 0. Then the value of f( 3 )
(A) can not be determined (B) is 2 (1  3 )

(C) is zero (D) is


2  32 
3

BANSAL CLASSES Private Ltd. ‘Bansal Tower’, A-10, Road No.-1, I.P.I.A., Kota-05 Page # 34
LIMIT, CONTINUITY AND DIFFERENTIABILITY OF FUNCTION

Q.11 Number of points of discontinuity of f(x) = [2x3 – 5] in [1, 2), is equal to


(where [x] denotes greatest integer less than or equal to x)
(A) 14 (B) 13 (C) 10 (D) 8

Q.12 The number of continuous functions f : R  R satisfying f (0) = 1 and f (2x) – f(x) = x , is
(A) 0 (B) 1 (C) 2 (D) Infinite

x
Q.13 Indicate all correct alternatives if, f (x) =  1 , then on the interval [0, ]
2
1 1
(A) tan ( f (x) ) & are both continuous (B) tan ( f (x) ) & are both discontinuous
f (x ) f (x )
1
(C) tan ( f (x) ) & f 1 (x) are both continuous (D) tan ( f (x) ) is continuous but is not
f (x )

Q.14 Which of the following functions are continuous at x = 0?


[Note : sgn x denotes signum function of x.]
   
(A) cos  sgn x   sgn x (B) cos  sgn x   sgn x
2  2 
   
(C) sin  sgn x   sgn x (D) sin  sgn x   sgn x
2  2 

Q.15 Which of the following function(s) is/are discontinuous at x = 0?


 
(A) f (x) = sin , x  0 and f (0) = 1 (B) g (x) = x sin   , x  0 and g (0) = .
2x x
|x| 1
(C) h (x) = , x  0 and h (0) = 1 (D) k (x) = , x  0 and k(0) = 0.
x 1  ecot x

SPECIAL DPP-7 [CONTINUITY]

sin x  x cos x
Q.1 Consider the function defined on [0, 1]  R, f (x) = if x  0 and f (0) = 0, then the
x2
function f (x)
(A) has a removable discontinuity at x = 0
(B) has a non removable finite discontinuity at x = 0
(C) has a non removable infinite discontinuity at x = 0
(D) is continuous at x = 0

e 2 x  (1  4 x )1 2
Q.2 If f (x) = for x  0, then f has
ln (1  x 2 )
(A) an irremovable discontinuity at x = 0
(B) a removable discontinuity at x = 0 and f (0) = – 4
(C) a removable discontinuity at x = 0 and f (0) = – 1/4
(D) a removable discontinuity at x = 0 and f (0) = 4

BANSAL CLASSES Private Ltd. ‘Bansal Tower’, A-10, Road No.-1, I.P.I.A., Kota-05 Page # 35
LIMIT, CONTINUITY AND DIFFERENTIABILITY OF FUNCTION

Q.3 Let function f be defined as f : R+  R+ and function g is defined as g : R+  R+.


x n 1 f ( x )  x 2
Functions f and g are continuous in their domain. Suppose function h(x) = Lim , x  0.
n  x n  g( x)
If h(x) is continuous in its domain then f(1).g(1) is equal to
1
(A) 2 (B) 1 (C) (D) 0
2
e x x cos x  x log e (1  x )  x
Q.4 Let f(x) = , x  0. If f(x) is continuous at x = 0, then f (0) is equal to
x2
(A) 0 (B) 1 (C) – 1 (D) 2

x[ x ]2 log (1 x ) 2 for  1  x  0



Q.5 Consider f(x) =  ln e x 2  2 x
   for0  x  1
tan x
where [ * ] & {*} are the greatest integer function & fractional part function respectively, then
(A) f(0) = ln2  f is continuous at x = 0 (B) f(0) = 2  f is continuous at x = 0
(C) f(0) = e  f is continuous at x = 0
2 (D) f has an irremovable discontinuity at x = 0

Paragraph for question nos. 6 to 8


2 32  
Consider a function, f (x) = ln (sin x  cos x ) for x   0, 
3  2
f (x)
Q.6 The value of Lim is
 32
x  
2   x
2 
2 2 34
(A) is (B) is 1 (C) is ( 2) (D) non existent
3 3

23
3   
 f (x )  x   0, 
 2   2

Q.7 The function g (x) is defined as g (x) =  23 .
3  
  2 f  x  2  
 e  
 
; x   , 
Then g (x) 2 2 
 
(A) is continuous at x = for g   0
2 2

(B) has a removable discontinuity at x =
2

(C) g (x) is discontinuous at x = and jump of discontinuity is equals to 2
2

(D) has a non-removable discontinuity at x =
2
BANSAL CLASSES Private Ltd. ‘Bansal Tower’, A-10, Road No.-1, I.P.I.A., Kota-05 Page # 36
LIMIT, CONTINUITY AND DIFFERENTIABILITY OF FUNCTION

Q.8 The range of g (x) is


 1  1   1   1 
(A) (0, 1] (B)  0, ln 2 (C)  ,1 (D)  0, ln 2   ,1
 2  2   2   2 

Q.9 Which of the following function(s) not defined at x = 0 has/have non-removable discontinuity at the
point x = 0 ?
1
1 1 ex  1 1
(A) f (x) = 1
(B) f(x) = arc tan (C) f (x) = 1 (D) f(x) =
1 2x x ex 1 n x

Q.10 Which of the following function(s) not defined at x = 0 has/have removable discontinuity at x=0?

(B) f(x)=cos  | sin x | 


1
(A) f(x) =
1  2cot x  x 
 1
(C) f(x) = x sin (D) f(x) =
x n x

 
Q.11 Which of the following function(s) will have atleast one real root in 0,  ?
 2
1
(A) f (x) = x – 1 – cosx (B) g(x) = + sin3x
2
3
1 x
(C) h(x) = (D) k(x) = 2
1 x x 1

e x  1  ax
, x0
 x2

 b, x0
Q.12 Let f (x) = 

 sin x
 2, x0
x

then
1
(A) f(x) is continuous at x = 0 if a = – 1, b  .
2
1
(B) f(x) is discontinuous at x = 0, if b  .
2
(C) f(x) has non-removable discontinuity at x = 0 if a  –1.
1
(D) f(x) has removable discontinuity at x = 0 if a = – 1, b  .
2

BANSAL CLASSES Private Ltd. ‘Bansal Tower’, A-10, Road No.-1, I.P.I.A., Kota-05 Page # 37
LIMIT, CONTINUITY AND DIFFERENTIABILITY OF FUNCTION

Q.13 Which of the following function(s) has removable type of discontinuity at x = 0?


1 sec2 2x csc x  1
(A) f (x) = (B) g (x) = x csc x (where csc x = cosec x)
4x 2
1
sin 5x 2 x2
(C) h (x) = (D) l (x) = (1  2 x )
x

Q.14 Consider the function f (x) = x 2 + [x]2 in [–2, 2] which of the following is correct?
(A) f is many one but not an even function in [– 2, 2].
(B) range of f contains 3 integers.
(C) f is periodic in [– 2, 2].
(D) f is discontinuous at all integers in (–2, 2).
Note: [y] denotes greatest integer less than or equal to y.

Q.15 Let S denotes the sum of an infinite geometric progression whose first term is the value of the function
sin x  ( 6) 
f (x) = at x = /6, if f (x) is continuous at x = /6 and whose common ratio is the limiting
3  2 cos x
sin( x )1 / 3 ln (1  3x )
as x  0. Find the value of 2S.
arc tan x  (e
value of the function g (x) = 2 1/ 3
5.x
 1)

SPECIAL DPP-8 [DERIVABILITY]

Q.1 State whether True or False:


1
(i) f (x) = Lim is continuous at x = 1.
n  1  n sin 2 x
 21 /(1  x )
(ii) The function f(x) = 2 if x  1 and f(1) = 1 is not continuous at x = 1.
(iii) There exists a continuous function f: [0, 1] onto [0, 10], but there exists no continuous function
g : [0, 1] onto (0, 10).
(iv) If f (x) is continuous in [0 , 1] and f(x) = 1 for all rational numbers in [0 , 1] then f 1  
2 equal to 1.

Q.2 The number of points where f(x) = [sin x + cos x] (where [ ] denotes the greatest integer function),
x  (0, 2) is not continuous is
(A) 3 (B) 4 (C) 5 (D) 6

f ( h )  f ( 2 h )
Q.3 Let f be differentiable at x = 0 and f ' (0) = 1. Then Lim =
h 0 h
(A) 3 (B) 2 (C) 1 (D) – 1

Q.4 The graph of function f contains the point P (1, 2) and Q(s, r). The equation of the secant line through
 s 2  2s  3 
P and Q is y =  s  1  x – 1 – s. The value of f ' (1), is
 
(A) 2 (B) 3 (C) 4 (D) non existent

BANSAL CLASSES Private Ltd. ‘Bansal Tower’, A-10, Road No.-1, I.P.I.A., Kota-05 Page # 38
LIMIT, CONTINUITY AND DIFFERENTIABILITY OF FUNCTION

Q.5 For what triplets of real numbers (a, b, c) with a 0 the function
x x 1
f(x) =  is differentiable for all real x ?
ax  bx  c otherwise
2

(A) {(a, 12a, a)  a  R, a 0 } (B) {(a, 12a, c)  a, c  R, a 0 }


(C) {(a, b, c)  a, b, c  R, a + b + c =1 } (D) {(a, 12a, 0)  a  R, a 0}

2  2 x  sin 2 x
Q.6 Lim is
x  (2 x  sin 2 x )e sin x
(A) equal to zero (B) equal to 1 (C) equal to  1 (D) non existent

Q.7 Let R be the set of real numbers and f : R  R, be a differentiable function such that
|f(x) – f(y)|  |x – y|3  x, y  R . If f(10) = 100, then the value of f(20) is equal to
(A) 0 (B) 10 (C) 20 (D) 100

3  x 2  2x 2  x  0

Let f (x) =  x 3  2
Q.8
 0x2
1 x 2x5
Statement-1: f (x) lies from f (–2) to f (2)  x  [–2, 5].
Statement-2: f (x) is not continuous in x  [–2, 5].
(A) Statement-1 is true, statement-2 is true and statement-2 is correct explanation for statement-1.
(B) Statement-1 is true, statement-2 is true and statement-2 is NOT the correct explanation for statement-1.
(C) Statement-1 is true, statement-2 is false.
(D) Statement-1 is false, statement-2 is true.

Q.9 Statement-1: The function | ln x | and ln x are both continuous for all x > 0.
Statement-2: Continuity of | f (x) |  continuity of f (x).
(A) Statement-1 is true, statement-2 is true and statement-2 is correct explanation for statement-1.
(B)Statement-1is true,statement-2 istrue and statement-2is NOTthe correct explanationforstatement-1.
(C) Statement-1 is true, statement-2 is false.
(D) Statement-1 is false, statement-2 is true.

2x  x n sin x
Q.10 Consider the function f(x) = Lim .
n  1  xn

 
Statement-1: The function f(x) vanishes for some x in  0,  .
 2


Statement-2: f   · f (0)  0 .
2
(A) Statement-1 is true, statement-2 is true and statement-2 is correct explanation for statement-1.
(B) Statement-1 is true, statement-2 is true and statement-2 is NOT the correct explanation for statement-1.
(C) Statement-1 is true, statement-2 is false.
(D) Statement-1 is false, statement-2 is true.

BANSAL CLASSES Private Ltd. ‘Bansal Tower’, A-10, Road No.-1, I.P.I.A., Kota-05 Page # 39
LIMIT, CONTINUITY AND DIFFERENTIABILITY OF FUNCTION

Q.11 f(x) = x[x] in  1  x  2 , where [x] is greatest integer  x then f(x) is :


(A) continuous at x = 0 (B) discontinuous x = 0
(C) not differentiable at x = 2 (D) differentiable at x = 2

Q.12 f(x) =1 + x [cosx] in 0 < x  /2 , where [ ] denotes greatest integer function then,
(A) It is continuous in 0 < x < /2 (B) It is differentiable in 0 < x < /2
(C) Its maximum value is 2 (D) It is not differentiable in 0 < x< /2

 x . n (cos x) x0
Q.13 If f(x) =  n 1  x2  then
 0 x0
(A) f is continuous at x = 0 (B) f is continuous at x = 0 but not differentiable at x = 0
(C) f is differentiable at x = 0 (D) f is not continuous at x = 0.

 x 3 ,x 1
Q.14 The function f(x) =   x 2   3x   13  is
        , x  1
  4   2  4
 
(A) continuous at x = 1 (B) differentiable at x = 1
(C) continuous at x = 3 (D) differentiable at x = 3

Q.15 Column-I contains 4functions and column-II contains comments w.r.t their continuityand differentiability
at x = 0. Note that column-I may have more than one matching options in column-II.
Column-I Column-II
(A) f (x) = [x] + | 1 – x | (P) continuous
[ ] denotes the greatest integer function
(B) g (x) = | x – 2 | + | x | (Q) derivability
(C) h (x) = [tan2x] (R) discontinuous
[ ] denotes the greatest integer function
x (3e1 x  4)
 x0
l (x) =  (2  e )
1x
(D) (S) non derivable

0 x0

SPECIAL DPP-9 [DERIVABILITY]

Q.1 If both f (x) & g(x) are differentiable functions at x = x0, then the function defined as,
h(x) = Maximum {f(x), g(x)}
(A) is always differentiable at x = x0
(B) is never differentiable at x = x0
(C) is differentiable at x = x0 when f(x0)  g(x0)
(D) cannot be differentiable at x = x0 if f(x0) = g(x0) .

Q.2  
If f (x) = Max. 1 | x |, Min.( x 2 ,1) then number of points where f (x) is non-differentiable is/are
(A) 5 (B) 4 (C) 3 (D) 2

BANSAL CLASSES Private Ltd. ‘Bansal Tower’, A-10, Road No.-1, I.P.I.A., Kota-05 Page # 40
LIMIT, CONTINUITY AND DIFFERENTIABILITY OF FUNCTION

Q.3 Let the function f, g and h be defined as follows :

1
x sin   for  1  x  1 and x  0
 x
f (x) = 

0 for x0

1
x 2 sin   for  1  x  1 and x  0
 x
g (x) = 

0 for x0
h (x) = | x |3 for – 1  x  1

Which of these functions are differentiable at x = 0?

(A) f and g only (B) f and h only (C) g and h only (D) none

x  b, x  0
Q.4 The function g (x) =  can be made differentiable at x = 0.
 cos x , x  0
(A) if b is equal to zero (B) if b is not equal to zero
(C) if b takes any real value (D) for no value of b

Q.5 Which one of the following functions is continuous everywhere in its domain but has atleast one point
where it is not differentiable?
|x|
(A) f (x) = x1/3 (B) f (x) = (C) f (x) = e–x (D) f (x) = tan x
x

Q.6 If the right hand derivative of f (x) = [x] tan x at x = 7 is k, then k is equal to
([y] denotes largest integer  y)
(A) 6 (B) 7 (C) – 7 (D) 49

Q.7 Let g : R  R be a differentiable function such that g (2) = – 40 and g ' (2) = – 5.
4
 g(2  x )  x 22
Then Lim  
 is equal to
x 0
 g(2) 
1
(A) e32 (B) e (C) (D) e–5
e

Q.8 Let f (x) = min(x3, x2)and g (x) = [x]2 + {x}2 , where [x] denotes the greatest integer and {x} denotes
the fractional part function. Then which of the following holds?
(A) f is continuous for all x. (B) g is discontinuous for all x  I.
(C) f is differentiable for all x  (1, ) (D) g is not differentiable for all x  I

BANSAL CLASSES Private Ltd. ‘Bansal Tower’, A-10, Road No.-1, I.P.I.A., Kota-05 Page # 41
LIMIT, CONTINUITY AND DIFFERENTIABILITY OF FUNCTION

Q.9 Column-I Column-II


f (3  h )  f ((3  h )
2 2
(A) If f (x) is derivable at x = 3 & f (3) = 2, then Limit equals (P) 0
h 0 2h 2
(B) Let f(x) be a function satisfying the condition f(x) = f(x) for all real x. (Q) 1
If f (0) exists, then its value is equal to
x
, x0

For the function f(x) =  1  e
1/ x
(C) , the derivative from the left, f(0) equals (R) 2
 0 , x0

(D) The number of points at which the function f(x) = max. {a  x, a + x, b}, (S) 3
  < x < , 0 < a < b cannot be differentiable is

Q.10 Column-I Column-II


(A) Consider the following functions defined from R to R (P) 1
(i) f(x) = x  x  1 (ii) g(x) = 2x  x

(iii) h(x) = x x (iv) k(x) = sin x (Q) 2


Number of functions which are not differentiable, is

(B) For a, b  R, if

Lim
2  x  3x  a  1  x  4x  b  10x
2 2 3

 2x 2  x  5
exists, (R) 3
x0 2
x
then (a2 + b2) is equal to

(C) If f(x) is derivable at x = 2 such that f(2) = 2 and f '(2) = 4, (S) 4

then the value of Lim


h0 h
1
2
 
ln f (2  h 2 )  ln f (2  h 2 ) is equal to

 
(D) Let f(x) = (x2 – 3x + 2) | (x3 – 6x2 + 11x – 6) | + sin x   . (T) 5
 4
Number of points at which the function f(x) is non-differentiable in [0, 2], is

 1 if | x |  1 1
A function f is defined as , f (x) = 
|x | 2
Q.11 1 . If f (x) is derivable at x = ,
a  b x2 if | x |  2
 2
then find (a – b).

a 2  ex ,   x  0


Q.12 Consider the function f(x) = x  2, 0x3
 b2
c  , 3 x  
 x
If f(x) is differentiable for every x  R, then find the number of ordered triplets (a, b, c) of real numbers.

BANSAL CLASSES Private Ltd. ‘Bansal Tower’, A-10, Road No.-1, I.P.I.A., Kota-05 Page # 42
LIMIT, CONTINUITY AND DIFFERENTIABILITY OF FUNCTION

SPECIAL DPP-10 [DERIVABILITY]

Q.1 Let f : R  R be a continuous onto function satisfying f(x) + f(– x) = 0, x R .


If f(–3) = 2 and f(5) = 4 in [– 5, 5], then the equation f(x) = 0 has
(A) exactly three real roots (B) exactly two real roots
(C) atleast five real roots (D) atleast three real roots

x
Q.2 The set of all points where the function f(x) = is differentiable is
1 x
(A) ( , ) (B) [ 0, ) (C) ( , 0)  (0, ) (D) (0, )

Q.3 The function f(x) = (x2  1) x2  3x + 2 + cos (x) is NOT differentiable at
(A)  1 (B) 0 (C) 1 (D) 2


 
n
 x 
 ax ( x  1) cot   px  2
2

 Lim  4 
Q.4 Let f (x) = n    x 
n , x (0, 1) (1, 2)
  cot  1
  4 
0 , x 1
If f (x) is differentiable for all x (0, 2) then (a2 + p2) equals
(A) 18 (B) 20 (C) 22 (D) 24

Q.5 Consider the functions f (x) = x2 – 2x and g (x) = – | x |


Statement-1: The composite function F (x) = f g ( x )  is not derivable at x = 0.
Statement-2: F ' (0+) = 2 and F ' (0–) = – 2.
(A) Statement-1 is true, statement-2 is true and statement-2 is correct explanation for statement-1.
(B)Statement-1is true,statement-2 istrue and statement-2is NOTthe correct explanationforstatement-1.
(C) Statement-1 is true, statement-2 is false.
(D) Statement-1 is false, statement-2 is true.

1
Q.6 Let f : R R be a continuous function defined by f (x) =
e  4e – 5 x
5x

1
Statement-1 : f(c) = , for some c R.
3 2
1
Statement-2 : 0 < f (x)  , for all x R.
4
(A) Statement-1 is true, statement-2 is false.
(B) Statement-1 is false, statement-2 is true.
(C) Statement-1 is true, statement-2 is true and statement-2 is correct explanation for statement-1.
(D) Statement-1 is true, statement-2 is true and statement-2 is NOT the correct explanation for statement-1.

BANSAL CLASSES Private Ltd. ‘Bansal Tower’, A-10, Road No.-1, I.P.I.A., Kota-05 Page # 43
LIMIT, CONTINUITY AND DIFFERENTIABILITY OF FUNCTION

Paragraph for question nos. 7 to 9


Suppose f, g and h be three real valued function defined on R.
1
Let f (x) = 2x + |x|, g (x) = (2x – |x| ) and h(x) = f g ( x ) 
3

 
1
Q.7 The range of the function k(x) = 1 + cos 1 (h ( x ))  cot 1 (h ( x )) is equal to

1 7  5 11 1 5  7 11
(A)  ,  (B)  ,  (C)  ,  (D)  , 
4 4 4 4  4 4 4 4 

Q.8 The domain of definition of the function l(x) = sin–1 f ( x )  g( x )  is equal to

3   3  3
(A)  ,   (B)  , 1 (C)   ,  (D)   ,
8   8  8 

Q.9 The function T(x) = f gf ( x )  + gf g( x )  , is


(A) continuous and differentiable in (– , ).
(B) continuous but not derivable  x  R.
(C) neither continuous nor derivable  x  R.
(D) an odd function.

 2 1
x cos x , x  0

Q.10 Let f(x) =  0 , x  0,
 2 1
 x sin x , x  0

then which of the following is(are) correct?
(A) f(x) is continuous but not differentiable at x = 0.
(B) f(x) is continuous and differentiable at x = 0.
(C) f (x) is continuous but not differentiable at x = 0.
(D) f (x) is discontinuous at x = 0.

Q.11 Which of the following statement(s) is/are correct?


f ( x 3 )  f (1)
(A) If f is differentiable at x = 1, then Lim  3f ' (1) .
x 1 x 1
f (ax)  f (bx) ab
(B) If f is differentiable at x = 0, then Lim  f ' (0) .
x0 cx c
x f ( x 0 )  x 0f ( x )
(C) If f is differentiable at x = x0, Lim = f(x0) – x0f '(x0).
x  x0 x  x0
(D) f : R  R is differentiable and f(x)  f(0)  x  [– 1, 1] then f '(0) = 0.

BANSAL CLASSES Private Ltd. ‘Bansal Tower’, A-10, Road No.-1, I.P.I.A., Kota-05 Page # 44
LIMIT, CONTINUITY AND DIFFERENTIABILITY OF FUNCTION

 4, 4  x  0
Q.12 Let f(x) =  2
x  4, 0x4

and g(x) = f  | x |   f ( x ) then which of the following statements is(are) correct ?



(A) f(x) is continuous and differentiable at x = .
2
(B) f(x) is continuous but non-differentiable at x = 0.
(C) g(x) is continuous as well as differentiable at x = 0.
(D) g(x) is continuous but not differentiable at x = 2.

Q.13 Let f (x) = 




1 2
 
 cos  ln (1  x ) , if x  0
x .

 0, if x  0
Then which of the following is(are) correct ?
(A) f (x) is continuous at x = 0.
(B) f (x) is differentiable at x = 0.
(C) f ' (x) is continuous at x = 0.
(D) f ' (x) is non-derivable at x = 0.

Q.14 For the quadratic polynomial f(x) = ax2 + bx + c where a, b, c  R and a > 0.
Match the entries of column-I with their corresponding entries in column-II
(Discriminant D = b2 – 4ac).
Column I Column II

(A) If D > 0, b < 0 and c > 0 then the number of points (P) 1
where the graph of y = | f (| x |) | is not derivable, is

(B) If D > 0, b < 0 and c < 0 then the number of points (Q) 2
where the graph of y = | f (| x |) | is not derivable, is

(C) If D < 0, b  0 and c > 0 then the number of points (R) 3


where the graph of y = | f (| x |) | is not derivable, is

(D) If D > 0, b = 0 and c < 0 then the number of points (S) 4


where the graph of y = | f (| x |) | is not derivable, is
(T) 5

 x
1  cos 1  cos 
Q.15 If Lim  2
is equal to the left hand derivative of e– | x | at x = 0,
x 0 m n
2 x
then find the value of (n – 10m).

BANSAL CLASSES Private Ltd. ‘Bansal Tower’, A-10, Road No.-1, I.P.I.A., Kota-05 Page # 45
LIMIT, CONTINUITY AND DIFFERENTIABILITY OF FUNCTION

EXERCISE-2

SECTION-A [LIMIT]

Q.1 Find the sum of an infinite geometric series whose first term is the limit of the function f(x) = 1  tan x
1  2 sin x
1 x
as x  /4 and whose common ratio is the limit of the function g(x) = as x  1.
(cos 1 x ) 2

8  x2 x2 x2 x2 
Q.2 Lim 1  cos  cos  cos cos 
x 0 x8 2 4 2 4

        
sin   4h   4 sin   3h   6 sin   2h   4 sin   h   sin
Q.3 Lim 3  3  3  3  3
h 0 4
h

 x2 x 3 
Q.4 Lim x 2  3 
x 
 x x 

(3x 4  2x 2 ) sin 1x  | x |3 5
Q.5 Lim
x 
| x |3  | x |2  | x | 1

 x2 1 
Q.6 Find a & b if : (i) x  
Lim  ax  b  = 0 (ii) xLim  x 2  x  1  ax  b  = 0
 x 1
  

2
e x  33x
Q.7 If Lim = ln K (where k  N), find K.
x 0  x2 
sin    sin x
 2 

1  1 1  ax  1 2 3
Q.8 If the Lim    exists and has the value equal to l, then find the value of   .
x 0 x  1  x 1  bx 
3
a l b

Q.9 Let f (x) = ax3 + bx2 + cx + d and g (x) = x2 + x – 2.


f (x) f (x) c2  d 2
If Lim = 1 and Lim = 4, then find the value of 2 .
x1 g ( x ) x  2 g( x ) a  b2

BANSAL CLASSES Private Ltd. ‘Bansal Tower’, A-10, Road No.-1, I.P.I.A., Kota-05 Page # 46
LIMIT, CONTINUITY AND DIFFERENTIABILITY OF FUNCTION

 x
1  cos1  cos 
 2  1
Q.10 If Lim = l, where l is finite and non zero number, then find value of  n   .
x 0 n
1  x n 1  l


 4 
  n3 1  
(1  n 1 ) 2
Q.11 Let L =  1  2  ; M =   
 n 3  1  and N =  1  2n 1
, then find the value of
n 3  n  n 2   n 1
–1 –1
L +M +N . –1

Q.12
2 sin {x}
(a) Let f(x) = . If f (5+) and f(3+) exists finitely and are not zero, then find the value of (a + b).
x 2  ax  b
[Note : {x} represents fractional part of x.]
5,   x  2
 3x  2,    x  1
(b) Let f(x) =  px , 2  x  4 and g(x) = x  1, 1  x   .
3x  q, 4  x   

If Lim f g( x )  = 5, then find the value of (2p + q).


x1

Q.13
tan 2x 1
 x   x  1  cos x  x
(i) Find Lim  tan  (ii) Find Lim
x 0  
x 1  4   x 
1/ x
x  1  x1/ x 
xc Lim  
(iii) Find Lim
x   x  c 
 4 then find c (iv) Find
  x 0  e 
 

1  2 x 
Q.14 Let f (x) = sin   and g (x) = | x | sgn (x).
1 x2 
 f (x )  a tan x  a sin x
If Lim   x 0
= Lim (a > 0) then find a.
x 0  g ( x )  tan x  sin x
[Note: [y] denotes greatest integer function less than or equal to yand sgn denotes signum function of x.]

Q.15 Let P(x) be a polynomial of degree '6' with leading coefficient unity and p(–x) = p(x) x R.
Also P(1)  32 + P2(2) + P(3)  52 = 0
sin(P( x ))
(i) Find the value of Lim .
x  2 ( x  2) tan( x  2)

 2 P( x ) 
x  
 2
 
Find the value of Lim  .
x 4
(ii)
x  1
x tan
x

BANSAL CLASSES Private Ltd. ‘Bansal Tower’, A-10, Road No.-1, I.P.I.A., Kota-05 Page # 47
LIMIT, CONTINUITY AND DIFFERENTIABILITY OF FUNCTION

      
If  and  ( < ) are the roots of the equation Lim cos –1  sin  tan –1 
tx  =
Q.16
t      6
 tx – 3tx  t – 1 – x   
2
 
then find the value of (8 + 2 – .

 
If n   0,  is a root of the equation (tan  + cot ) = n, n  2, then find Limsin  n  cos  n  .
n
Q.17
 4 n  

Q.18 Let f(x) be a quadratic function such that f(0) = f(1) = 0 and f(2) = 1.
 
cos  cos 2 x 
If Lim 2  equals k, k  N, then find k.
x0 2
f (x)

1
2 x 2 n sin x
Q.19 Let f (x) = Lim x then find
n  1 x 2n

(a) Lim x f ( x ) , (b) Lim f ( x ) , (c) Lim f ( x ) , (d) Lim f ( x )


x  x1 x0 x 

Q.20 Using Sandwich theorem, evaluate


 1 1 1 1 
(a) Lim     ...........  
n   
 n
2
n 1
2
n 2
2
n  2n 
2

1 2 n
(b) Lim + + ......... +
n  1 n2 2  n 2 n  n2

SECTION-B [CONTINUITY]

Q.1 Find all possible values of a and b so that f (x) is continuous for all x  R if
| ax  3 | if x  1

 | 3x  a | if  1  x  0
f (x) =  b sin 2x
  2b if 0  x  
 x
cos2 x  3 if x  


tan6x
 6 tan5x
5
if 0x 2
Q.2 The function f(x) =  b2 if x 2
  a tan x 
1 cosx  b  if 
2
x
Determine the values of 'a' & 'b' , if f is continuous at x = /2.

BANSAL CLASSES Private Ltd. ‘Bansal Tower’, A-10, Road No.-1, I.P.I.A., Kota-05 Page # 48
LIMIT, CONTINUITY AND DIFFERENTIABILITY OF FUNCTION

Q.3 Find the number of points of discontinuity of the function f(x) = [5x] + {3x} in [0, 5]
where [y] and {y} denote largest integer less than or equal to y and fractional part of y respectively.

Q.4 Let f (x) = tan x  1  4  tan x . If g (x) = [f (x)], then find sum of all possible values of tan x
 
in  0,  where g (x) is discontinuous.
 2
[Note: [y] denotes the greatest integer function less than or equal to y.]

 3 sin x cos x 2
Q.5 Consider, f(x) = 12 – 2 , x  R.
If g(x) = sgn (x + 2px + 5p + f (t) ) is continuous for all x  R then find the sum of all possible integral
2

values of 'p'.

ax 2  bx  c  e nx
Q.6 A function f : R  R is defined as f (x) = Lim where f is continuous on R. Find the
n  1  c · e nx
values of a, b and c.

Q.7 Find the number of different real values of b for which the function

  1
 sin x  2 
  , x  1
 2x  a 2

 1
f(x) = b, x
 2

 2 x 2  3x  1 1
 , x
 c  2x 2

1
is continuous at x = .
2

a sin x  a tan x
Q.8 f (x) = for x > 0
tan x  sin x
ln (1  x  x 2 )  ln (1  x  x 2 )
= for x < 0, if f is continuous at x = 0, find 'a'
sec x  cos x

now if g (x) = ln  2   cot(x – a) for x  a, a0, a>0. If g is continuous at x=a then show
x
 a
–1
that g(e ) = – e.

BANSAL CLASSES Private Ltd. ‘Bansal Tower’, A-10, Road No.-1, I.P.I.A., Kota-05 Page # 49
LIMIT, CONTINUITY AND DIFFERENTIABILITY OF FUNCTION

cosec x 
(sin x  cos x ) ; x0
 2


 a ; x0
Q.9 Let f (x) =  3
 1 2
x
 ex e x e 
 3
; 0x

2 2
x
ae x  be


If f (x) is continuous at x = 0, find the value of a 2  b 2 . 

 tan[ x ]
a  , x0
 x
Q.10 Let f (x) = 3, x0
  x  tan x 
b   , x  0
  x3 

 r
a
If f (x) is continuous at x = 0 then find the value of    .
r 0  b 
[Note: [k] denotes the largest integer less than or equal to k.]

Q.11 Let P(x) = x10 + a1x8 + a2x6 + a3x4 + a4x2 – 1 be a polynomial with real coefficients. If P(1) = 1,
P(2) = – 1 then find the minimum number of real zero's of P(x).

Q.12 (a) If g : [a, b] onto [a, b] is continuous show that there is some c  [a, b] such that g (c) = c.
(b) Let f be continuous on the interval [0, 1] to R such that f (0) = f (1). Prove that there exists a
 1  1
point c in 0,  such that f (c) = f  c  
 2  2

SECTION-C [DERIVABILITY]

x2
  for x  0
Q.1 If the function f (x) defined as f (x) =  2 is continuous but not derivable at x = 0
 n 1
x sin for x  0
x
then find the range of n.

tan x  tan y
Q.2 Let g(y) = Lim and f(x) = x2. If h(x) = Min. f ( x ), g( x )  , find the number
xy x  x
1   1   · tan x tan y
y  y
of points where h(x) is non-derivable.

BANSAL CLASSES Private Ltd. ‘Bansal Tower’, A-10, Road No.-1, I.P.I.A., Kota-05 Page # 50
LIMIT, CONTINUITY AND DIFFERENTIABILITY OF FUNCTION

Q.3 Let f (x) = | x – 1 | + sgn (sin x). If L and M denote the number of points on x  (– 1, ] where f (x)
is discontinuous and non-differentiable respectively, then find (L+ M).
[Note: sgn x denotes signum function of x.]
  1  1x 
Q.4 Let f(x) = xe 
x  ; x  0 , f(0) = 0, test the continuity & differentiability at x = 0

ax 2  b if x  1
Q.5 If f(x) =   1 is derivable at x = 1. Find the values of a & b.
 if x  1
 | x |

Q.6 Consider the function f defined as f(x) = | x |  1  x  R and another function g(x) such that
g(x) = (fof) (x). Find number of points where g(x) is non-differentiable.

1 ,  2  x  0
Q.7 Let f(x) be defined in the interval [– 2, 2] such that f(x) =  &
 x 1 , 0x2
g(x) = f(x) + f(x). Test the differentiability of g(x) in ( 2, 2).

Q.8 Examine for continuity & differentiability the points x = 1 & x = 2, the function f defined by
x [x] , 0x2
f(x) =  where [x] = greatest integer less than or equal to x.
(x  1) [x] , 2  x  3

 ln (1  x )  ln (1  x )
x. , x  (1,0)
Q.9 Let a1 and a2 be two values of a for which f (x) =  sec x  cos x
(a 2  3a  1) x  x 2 , x  [0, )
is differentiable at x = 0, then find the value of (a12 + a22).

ax( x  1)  b, x  1

Q.10 Let g(x) = x  2, 1 x  3
px  qx  2, x  3
2

is continuous for all x  R except at x = 1 but | g (x) | is differentiable everywhere and g'(x)
is continuous at x = 3. Find the value of | a + p + b + q |.

Q.11(a) Let f (x) be a differentiable function on R such that


2 f (x + y) + f (x – y) = 3 f (x) + 3 f (y) + 2xy for all x, y  R.
If f ' (0) = 0 then find the value of f (3).
x  y 1
(b) Let f be a differentiable function satisfying the functional rule f(xy) = f(x) + f(y) +  x, y > 0
xy

and f '(1) = 2. Find the value of f (e100 ) . 
Note : [k] denotes the greatest integer less than or equal to k.

Q.12(a) Let g(x) = (x5 + 1) | x2 – 4x – 5 | + sin | x | + cos ( | x – 1| )  x  R then find the number of points of
non-differentiability of g(x) on R.
(b) Let f (x) = x2 – x + k – 2, k  R. If the complete set of values of k for which y = f | x | is
non-derivable at 5 distinct points is (a, b) then find the value of 8(b – a).

BANSAL CLASSES Private Ltd. ‘Bansal Tower’, A-10, Road No.-1, I.P.I.A., Kota-05 Page # 51
LIMIT, CONTINUITY AND DIFFERENTIABILITY OF FUNCTION

EXERCISE-3
SECTION-A
(JEE-MAIN Previous Year's Questions)
LIMIT
1  cos 2x
Q.1 Lim is [AIEEE 2002]
x 0 2x
(1)  (2) – 1 (3) zero (4) does not exist

x f ( 2)  2f ( x )
Q.2 Let f(2) = 4 and f '(2) = 4. Then Lim is given by [AIEEE 2002]
x 2 x2
(1) 2 (2) – 2 (3) – 4 (4) 3

x
 2 
Q.3 lim  x  5x  3  = [AIEEE 2002]
x   x 2  x  3 
 
(1) e4 (2) e2 (3) e3 (4) e

log x n  [ x ]
Q.4 lim , nN, (where [x] denotes greatest integer less than or equal to x) [AIEEE-2002]
x  [x]
(1) Has value – 1 (2) Has value 0 (3) Has value 1 (4) Does not exist

log (3  x )  log (3  x )
Q.5 If xlim
0
= k, the value of k is - [AIEEE 2003]
x
2 1 2
(1) – (2) 0 (3) – (4)
3 3 3

Q.6 Let f(a) = g(a) = k and their nth derivatives f n (a), gn(a) exist and are not equal for some n.
f ( a ) g ( x )  f (a )  g ( a ) f ( x )  g (a )
Further if xlim
a
= 4 then the value of k is [AIEEE 2003]
g (x )  f (x )
(1) 0 (2) 4 (3) 2 (4) 1

  x 
1  tan   [1  sin x ]
lim   2 
Q.7  is [AIEEE 2003]
x
2   x 
1  tan  2  [  2 x ]
3

  
1 1
(1)  (2) (3) 0 (4)
8 32

BANSAL CLASSES Private Ltd. ‘Bansal Tower’, A-10, Road No.-1, I.P.I.A., Kota-05 Page # 52
LIMIT, CONTINUITY AND DIFFERENTIABILITY OF FUNCTION

2x
 a b 
Q.8 If xlim 1   2  = e2, then the values of a and b, are
 
[AIEEE 2004]
x x 
(1) a  R, b  R (2) a = 1, b  R (3) a  R, b = 2 (4) a  1 and b  2

1  cos (ax 2  bx  c)
Q.9 Let  and  be the distinct roots of ax2 + bx + c = 0, then Lim is equal to
x  ( x  )2

a2  a2 1
(1) ( – )2 (2) 0 (3) ( – )2 (4) ( – )2
2 2 2
[AIEEE 2005]

f (3x ) f (2x )
Q.10 Let f : R R be a positive increasing function with Lim = 1. Then Lim is equal to
x  f ( x ) x  f ( x )

2 3
(1) 1 (2) (3) (4) 3 [AIEEE 2010]
3 2

 1  cos{2( x  2)} 
Q.11 Lim   [AIEEE 2011]
x 2  x  2 
 
1
(1) does not exist (2) equals 2 (3) equals – 2 (4) equals
2

(1  cos 2 x ) (3  cos x )
Q.12 Lim is equal to
x 0 x tan 4 x
1 1
(1) (2) 1 (3) 2 (4) [JEE (Main) 2013]
2 4

sin ( cos2 x )
Q.13 Lim is equal to
x 0 x2

(1)  (2) (3) 1 (4) –  [JEE (Main) 2014]
2

(1  cos 2 x )(3  cos x )


Q.14 Lim is equal to [JEE (Main) 2015]
x 0 x tan 4 x
1
(1) 2 (2) (3) 4 (4) 3
2

BANSAL CLASSES Private Ltd. ‘Bansal Tower’, A-10, Road No.-1, I.P.I.A., Kota-05 Page # 53
LIMIT, CONTINUITY AND DIFFERENTIABILITY OF FUNCTION

1
Q.15 Let p = Lim (1  tan 2 x ) 2 x then log p is equal to [JEE (Main) 2016]
x 0

1 1
(1) (2) 2 (3) 1 (4)
4 2

cot x  cos x
Q.16 Lim equals [JEE (Main) 2017]
x
 (   2 x )3
2

1 1 1 1
(1) (2) (3) (4)
24 16 8 4

Q.17 For each t  R, let [t] be the greatest integer less than or equal to t.
1 2 15  
Then Lim x        ...........    
x 0 x x  x 
(1) is equal to 120 (2) does not exist (in R)
(3) is equal to 0 (4) is equal to 15 [JEE (Main) 2018]

CONTINUITY AND DIFFERENTIABILITY


  1  1 
  | x| x 
Q.1 If f(x) = x e  , x  0
then f(x) is [AIEEE 2003]
 0 , x0
(1) discontinuous everywhere
(2) continuous as well as differentiable for all x
(3) continuous for all x but not differentiable at x = 0
(4) neither differentiable nor continuous at x = 0

1  tan x      
Q.2 Let f(x) =
4x  
,x ,x 0, 2  . If f(x) is continuous in 0, 2  , then f  4  is
4      
(1) 1 (2) 1/2 (3) – 1/2 (4) –1 [AIEEE 2004]

Q.3 If f is a real-valued differentiable function satisfying | f(x) – f (y) |  (x – y)2, x, y  R and


f(0) = 0, then f(1) equals
(1) –1 (2) 0 (3) 2 (4) 1 [AIEEE 2005]

1
Q.4 Suppose f(x) is differentiable at x = 1 and Lim f(1 + h) = 5, then f '(1) equals
h0 h
(1) 3 (2) 4 (3) 5 (4) 6 [AIEEE 2005]

x
Q.5 The set of points where f (x) = is differentiable is
1 | x |
(1) (– , – 1)  (–1, ) (2) (– , )
(3) (0, ) (4) (– , 0)  (0, ) [AIEEE 2006]

BANSAL CLASSES Private Ltd. ‘Bansal Tower’, A-10, Road No.-1, I.P.I.A., Kota-05 Page # 54
LIMIT, CONTINUITY AND DIFFERENTIABILITY OF FUNCTION

1 2
Q.6 The function f : R\{0}  R given by f (x) =  2x can be made continuous at x = 0 by
x e 1
defining f(0) as
(1) 2 (2) –1 (3) 0 (4) 1 [AIEEE 2007]

Q.7 Let f : R  R be a function defined by f (x) = Min {x + 1, |x| + 1}. Then which of the following
is true ?
(1) f(x)  1 for all x  R (2) f(x) is not differentiable at x = 1
(3) f(x) is differentiable everywhere (4) f(x) is not differentiable at x = 0
[AIEEE 2007]
 1
( x  1) sin if x  1
Q.8 Let f(x) =  x 1 . Then which one of the following is true ?
 if x  1
 0
(1) f is differentiable at x = 0 and at x = 1
(2) f is differentiable at x = 0 but not at x = 1
(3) f is differentiable at x = 1 but not at x = 0
(4) f is neither differentiable at x = 0 nor at x = 1 [AIEEE 2008]

Q.9 Let f(x) = x | x | and g(x) = sin x.


Statement – 1 : gof is differentiable at x = 0 and its derivative is continuous at that point.
Statement – 2 : gof is twice differentiable at x = 0.
(1) Statement -1 is true, Statement -2 is true; Statement -2 is a correct explanation for Statement -1
(2) Statement -1 is true, Statement -2 is true; Statement -2 is not a correct explanation for Statement-1.
(3) Statement -1 is true, Statement -2 is false.
(4) Statement -1 is false, Statement -2 is ture. [AIEEE 2009]

 sin(p  1) x  sin x
 , x0
 x
Q.10 The value of p and q for which the function f(x) =  q , x0

 xx  x , x 0
2

 x3/ 2
is continuous for all x in R, are
1 3 5 1
(1) p = ,q=– (2) p = ,q=
2 2 2 2
3 1 1 3
(3) p = – ,q= (4) p = ,q= [AIEEE 2011]
2 2 2 2

k x  1 , 0  x  3
Q.11 If the function g(x) =  mx  2 , 3  x  5 is differentiable, then the value of k + m is

10 16
(1) (2) 4 (3) 2 (4)
3 5
[JEE (Main) 2015]
BANSAL CLASSES Private Ltd. ‘Bansal Tower’, A-10, Road No.-1, I.P.I.A., Kota-05 Page # 55
LIMIT, CONTINUITY AND DIFFERENTIABILITY OF FUNCTION

Q.12 For x  R, f(x) = | log 2 – sin x | and g(x) = f  f ( x )  , then


(1) g is differentiable at x = 0 and g'(0) = – sin (log 2)
(2) g is not differentiable at x = 0
(3) g'(0) = cos (log 2)
(4) g'(0) = – cos (log 2) [JEE (Main) 2016]

Q.13 Let S = {t  R : f(x) = |x – | · (e| x | –1) sin | x | is not differentiable at t}. Then the set S is equal to
(1) {} (2) {0, } (3)  (an empty set) (4) {0}
[JEE (Main) 2018]

SECTION-B
(JEE-ADVANCE Previous Year's Questions)

sin( cos 2 x )
Q.1 Lim equals [ JEE 2001, Screening]
x 0 x2

(A) – (B)  (C) (D) 1
2
Q.2 The left hand derivative of , f (x) = [ x ] sin ( x) at x = k , k an integer is
where [ ] denotes the greatest function. [JEE 2001 (Screening)]
(A) ( 1)k(k  1) (B) ( 1)k  1(k  1) (C) ( 1)k k  (D) ( 1)k  1 k 

Q.3 Let f : R  R be a function defined by, f (x) = max [x, x3]. The set of all points where
f (x) is NOT differentiable is
(A) { 1 , 1} (B) { 1 , 0} (C) {0 , 1} (D) { 1 , 0 , 1}
[JEE 2001 (Screening)]

Q.4 Which of the following functions is differentiable at x = 0?


(A) cos (x) + x (B) cos (x)  x (C) sin (x) + x (D) sin (x)  x
[JEE 2001 (Screening)]

Q.5 Let   R. Prove that a function f : R  R is differentiable at  if and only if there is a function
g : R  R which is continuous at  and satisfies f(x) – f() = g(x) (x –) for all x R.
[JEE 2001, (mains) 5 out of 100]

(cos x  1)(cos x  e x )
Q.6 The integer n for which Lim is a finite non-zero number is
x 0 xn
(A) 1 (B) 2 (C) 3 (D) 4
[JEE 2002 (screening), 3]

 tan 1 x if | x|  1

Q.7 The domain of the derivative of the function f (x) =  1 is
 2 (| x|1) if | x|  1
(A) R – {0} (B) R – {1} (C) R – {–1} (D) R – {–1, 1}
[JEE 2002 (screening), 3]

BANSAL CLASSES Private Ltd. ‘Bansal Tower’, A-10, Road No.-1, I.P.I.A., Kota-05 Page # 56
LIMIT, CONTINUITY AND DIFFERENTIABILITY OF FUNCTION

1/x
 f(1  x) 
Q.8 Let f : R  R be such that f (1) = 3 and f (1) = 6. The Limit   equals
x 0  f(1) 
(A) 1 (B) e1/2 (C) e2 (D) e3
[JEE 2002 (screening), 3]

x  a if x  0 x  1 if x  0
Q.9 f (x) =  and g (x) = 
| x  1| if x  0 ( x  1 )  b if x  0
2

Where a and b are non negative real numbers. Determine the composite function gof. If (gof) (x) is
continuous for all real x, determine the values of a and b. Further, for these values of a and b, is gof
differentiable at x = 0? Justify your answer. [JEE 2002, 5 out of 60]

sin(n x )[(a  n )n x  tan x ]


Q.10 If Lim  0 (n > 0) then the value of 'a' is equal to
x 0 x2
1 n2 1
(A) (B) n2 + 1 (C) (D) None
n n
[JEE 2003 (screening)]

Q.11 If a function f : [ –2a , 2a]  R is an odd function such that f (x) = f (2a – x) for x  [a, 2a] and the left
hand derivative at x = a is 0 then find the left hand derivative at x = – a.
[JEE 2003(Mains) 2 out of 60]

Q.12 Evaluate: Lim  2 (n  1) cos 1  1   n  . [JEE ' 2004, 2 out of 60]


n    n 

Q.13(a) The function given by y = | x | 1 is differentiable for all real numbers except the points
(A) {0, 1, –1} (B) ± 1 (C) 1 (D) – 1
[JEE 2005 (Screening), 3]
(b) If | f(x1) – f(x2) |  (x1 – x2) , for all x1, x2  R. Find the equation of tangent to the curve y = f (x) at the
2

point (1, 2). [JEE 2005 (Mains), 2]

Q.14 If f (x) = min. (1, x2, x3), then [JEE 2006, 5]


(A) f (x) is continuous  x  R
(B) f x   0 ,  x > 1
(C) f(x) is not differentiable but continuous  xR
(D) f(x) is not differentiable for two values of x

( x  1) n
Q.15 Let g(x) = ; 0 < x < 2, m and n are integers, m  0, n > 0 and let p be the left hand
ln cos m ( x  1)
derivative of | x – 1 | at x = 1. If Lim g(x) = p, then
x 1
(A) n = 1, m = 1 (B) n = 1, m = –1 (C) n = 2, m = 2 (D) n > 2, m = n
[JEE 2008, 3]

BANSAL CLASSES Private Ltd. ‘Bansal Tower’, A-10, Road No.-1, I.P.I.A., Kota-05 Page # 57
LIMIT, CONTINUITY AND DIFFERENTIABILITY OF FUNCTION

x2
a  a2  x2 
Q.16 Let L  Lim 4 , a  0 . If L is finite, then [ JEE ' 2009, 4]
x 0 x4
1 1
(A) a = 2 (B) a = 1 (C) L = (D) L =
64 32

1
Q.17(a) If Lim [1  x ln (1  b 2
)] x = 2b sin2, b > 0 and   (–, ], then the value of  is
x 0

   
(A)  (B)  (C)  (D) 
4 3 6 2

  
 x  2 , x
2
 
(b) If f ( x )   cos x, x0 , then
 2
x  1, 0  x 1
ln x , x 1


(A) f (x) is continuous at x = (B) f (x) is not differentiable at x = 0
2
3
(C) f (x) is differentiable at x = 1 (D) f (x) is differentiable at x =
2

(c) Let f : R  R be a function such that f (x + y) = f (x) + f (y),  x, y  R.


If f(x) is differentiable at x = 0, then
(A) f (x) is differentiable onlyin a finite interval containing zero.
(B) f (x) is continuous  x  R.
(C) f '(x) is constant  x  R.
(D) f (x) is differentiable except at finitely many points.

bx
(d) Let f : (0, 1)  R be defined by f (x) =
1  bx
where b is a constant such that 0 < b < 1. Then
1
(A) f is not invertible on (0, 1) (B) f  f –1 on (0, 1) and f '(b) =
f ' (0)
1
(C) f = f –1 on (0, 1) and f '(b) = (D) f –1 is differentiable on (0, 1)
f ' (0)
[JEE 2011, 3+4+4+4]

BANSAL CLASSES Private Ltd. ‘Bansal Tower’, A-10, Road No.-1, I.P.I.A., Kota-05 Page # 58
LIMIT, CONTINUITY AND DIFFERENTIABILITY OF FUNCTION

Q.18
 2 
x cos , x0
(a) Let f(x) =  x , x  R,
0, x0
then f is
(A) differentiable both at x = 0 and at x = 2.
(B) differentiable at x = 0 but not differentiable at x = 2.
(C) not differentiable at x = 0 but differentiable at x = 2.
(D) differentiable neither at x = 0 nor at x = 2.
 x2  x 1 
(b) If Lim   ax  b  = 4, then
x 
 x 1 
(A) a = 1, b = 4 (B) a = 1, b = – 4 (C) a = 2, b = – 3 (D) a = 2, b = 3

(c) Let  (a) and  (a) be the roots of the equation  3



1 a 1 x2    
1 a 1 x  6

1  a  1 = 0,
where a > – 1. Then Lim (a ) and Lim (a ) are
a 0  a 0 

5 1 7 9
(A) and 1 (B) and –1 (C) and 2 (D) and 3
2 2 2 2

(d) For every integer n, let an and bn be real numbers. Let function f : R  R be given by
a  sin x , for x [2n , 2n  1]
f (x) =  n , for all integers n.
 n
b  cos x , for x  (2n  1, 2n )
If f is continuous, then which of the following hold(s) for all n?
(A) an – 1 – bn – 1 = 0 (B) an – bn = 1
(C) an – bn + 1 = 1 (D) an – 1 – bn = – 1 [JEE 2012, 3+3+3+4]

Q.19 For every pair of continuous functions f, g : [0, 1]  R such that


max {f(x) : x  [0,1]} = max {g(x) : x  [0,1]},
the correct statement(s) is/are
(A)  f (c)  2 + 3f(c) =  g (c)  2 + 3g(c) for some c  [0, 1].

(B)  f (c)  2 + f(c) =  g (c)  2 + 3g(c) for some c  [0, 1].

(C)  f (c)  2 + 3f(c) =  g (c)  2 + g(c) for some c  [0, 1].

(D)  f (c)  2 =  g (c)  2 for some c  [0, 1]. [JEE (Advanced) 2014, 3]

Q.20 Let f : R  R and g : R  R be respectively given by f (x) = | x | + 1 and g (x) = x2 + 1.


Define h : R  R by
max f ( x ), g ( x ) if x  0
h (x) = 
min . f ( x ), g ( x ) if x  0
.

The number of points at which h (x) is not differentiable is [JEE (Advanced) 2014, 3]

BANSAL CLASSES Private Ltd. ‘Bansal Tower’, A-10, Road No.-1, I.P.I.A., Kota-05 Page # 59
LIMIT, CONTINUITY AND DIFFERENTIABILITY OF FUNCTION

1 x
  ax  sin( x  1)  a 1  x 1
Q.21 The largest value of the non-negative integer a for which Lim   = is
x 1  x  sin( x  1)  1  4
[JEE (Advanced) 2014, 3]

Q.22 Let f1 : R  R, f2 : [0, )  R, f3 : R  R and f4 : R  [0, ) be defined by


| x |, if x  0
f1(x) =  x
e , if x  0
f2(x) = x2,
sin x , if x  0 f  f ( x ) , if x  0
f3(x) =  and f4(x) = f 2  f1 ( x )   1, if x  0
x , if x  0 2 1
List-I List-II
P. f4 is 1. onto but not one-one
Q. f3 is 2. neither continuous nor one-one
R. f2of1 is 3. differentiable but not one-one
S. f2 is 4. continuous and one-one
Codes:
P Q R S
(A) 3 1 4 2
(B) 1 3 4 2
(C) 3 1 2 4
(D) 1 3 2 4 [JEE (Advanced) 2014, 3]

Q.23 Let g : R  R be a differentiable function with g (0) = 0, g ' (0) = 0 and g ' (1)  0.
 x
 g( x ), x  0
Let f (x) = | x |
0, x0

and h (x) = e|x| for all x  R. Let (f o h)(x) denote f h ( x )  and (h o f ) (x) denote h f ( x )  .
Then which of the following is(are) true?
(A) f is differentiable at x = 0 (B) h is differentiable at x = 0
(C) f o h is differentiable at x = 0 (D) h o f is differentiable at x = 0
[JEE (Advanced) 2015, 4]

   
Q.24 Let f(x) = sin  sin sin x   for all x  R and g(x) = sin x for all x  R. Let (fog)(x) denote
6 2  2
f (g(x)) and (g o f )(x) denote g (f(x)). Then which of the following is (are) true?
 1 1   1 1 
(A) Range of f is  ,  (B) Range of f o g is  , 
 2 2  2 2
f (x) 
(C) Lim = (D) There is an x  R such that (g o f )(x) = 1
x0 g(x ) 6
[JEE (Advanced) 2015, 4]

BANSAL CLASSES Private Ltd. ‘Bansal Tower’, A-10, Road No.-1, I.P.I.A., Kota-05 Page # 60
LIMIT, CONTINUITY AND DIFFERENTIABILITY OF FUNCTION

 ecos( )  e 
n

Q.25 Let m and n be two positive integers greater than 1. If Lim      e  then the value of
 0  
  2
m

m
is
n
[JEE (Advanced) 2015, 4]

 3ax 2  2, x 1
Q.26 Let a and b be real numbers such that the function f(x) = bx  a 2 , x 1

is differentiable for all x  R. Then possible value(s) of a is (are)
(A) 1 (B) 2 (C) 3 (D) 4
[JEE (Advanced) 2015, MTC, 2]

x 2 sin( x )
Q.27 Let ,   R be such that Lim = 1. Then 6 ( + ) equals
x 0 x  sin x
[JEE (Advanced) 2016, 3]

 1   1 
Q.28 Let f :  , 2  R and g :  , 2  R be functions defined by f(x) = [x 2 – 3] and
 2   2 
g(x) = | x | f(x) + |4x – 7| f(x), where [y] denotes the greatest integer less than or equal to y for y  R.
Then
 1 
(A) f is discontinuous exactly at three points in  , 2
 2 
 1 
(B) f is discontinuous exactly at four points in  , 2
 2 
 1 
(C) g is NOT differentiable exactly at four points in   , 2 
 2 
 1 
(D) g is NOT differentiable exactly at five points in   , 2  [JEE (Advanced) 2016, 4]
 2 

Q.29 Let a, b  R and f : R  R be defined by f(x) = a cos (| x3 – x |) + b | x | sin (| x3 + x |). Then f is


(A) differentiable at x = 0 if a = 0 and b = 1
(B) differentiable at x = 1 if a = 1 and b = 0
(C) NOT differentiable at x = 0 if a = 1 and b = 0
(D) NOT differentiable at x = 1 if a = 1 and b = 1 [JEE (Advanced) 2016, 4]

Q.30 Let [x] be the greatest integer less than or equal to x. Then, at which of the following point(s) the function
f(x) = x cos   ( x  [ x ])  is discontinuous?
(A) x = 0 (B) x = 1 (C) x = 2 (D) x = – 1
[JEE (Advanced) 2017, 4]

BANSAL CLASSES Private Ltd. ‘Bansal Tower’, A-10, Road No.-1, I.P.I.A., Kota-05 Page # 61
LIMIT, CONTINUITY AND DIFFERENTIABILITY OF FUNCTION

1  x  1 | 1  x |   1 
Q.31 Let f(x) = cos   for x  1. Then
|1 x |  1 x 
(A) Lim f ( x ) does not exist (B) Lim f ( x ) does not exist
x 1 x 1

(C) Lim f ( x ) = 0 (D) Lim f ( x ) = 0


x 1 x 1
[JEE (Advanced) 2017, 4]

Q.32 Let f : R  R be a differentiable function with f (0) = 1 and satisfying the equation
f (x + y) = f (x) f '(y) + f '(x) f (y) for all x, y  R.
Then, the value of log e f (4)  is______. [JEE (Advanced) 2018, 3]

  
Q.33 Let f1 : R  R, f2 :   ,   R, f3 : (–1, e/2 – 2)  R and f4 : R  R be functions defined by
 2 2
 x 2 
(i) f1(x) = sin  1  e ,
 

 | sin x |
 if x  0
(ii) f2(x) =  tan 1 x , where the inverse trigonometric function tan–1 x assumes values in
 1 if x  0

  
 , ,
 2 2
(iii) f3(x) =  sin log e ( x  2)  , where, for t R, [t] denotes the greatest integer less than or equal
to t,
 2 1
x sin  if x  
(iv) f4(x) =  x
 0 if x  0
List-I List-II

(P) The function f1 is (1) NOT continuous at x = 0

(Q) The function f2 is (2) continuous at x = 0 and NOT differentiable at x = 0

(R) The function f3 is (3) differentiable at x = 0 and its derivative is NOT


continuous at x = 0

(S) The function f4 is (4) differentiable at x = 0 and its derivative is


continuous at x = 0
The correct option is:
(A) P  2; Q  3; R 1; S 4 (B) P  4; Q  1; R 2; S 3
(C) P  4; Q  2; R 1; S 3 (D) P  2; Q  1; R 4; S 3
[JEE (Advanced) 2018, 3]

BANSAL CLASSES Private Ltd. ‘Bansal Tower’, A-10, Road No.-1, I.P.I.A., Kota-05 Page # 62
LIMIT, CONTINUITY AND DIFFERENTIABILITY OF FUNCTION

EXERCISE-4
SECTION-A
(CBSE Previous Year's Questions)
Q.1 Show that the function f (x) = 2x – | x | is continuous at x = 0. [CBSE 2002]

2 x  1, x  0
Q.2 Discuss the continuity of f (x) =  at x = 0 [CBSE 2002]
2 x  1, x  0

 x
 , x0
Q.3 Discuss the continuity of function f (x) = | x | 2x 2 , at x = 0. [CBSE 2005]
2, x0
Q.4 Find the relationship between a and b so that the function f defined by
ax  1, if x  3
f (x) =  is continuous at x = 3. [CBSE 2005, 2011]
bx  3, if x  3

m( x 2  2 x ), x  0
Q.5 Find the value of m such that the function f (x) =  is continuous at x = 0.
cos x , x0
[CBSE 2006]
Q.6 If f (x), defined by the following, is continuous at x = 0, find the value of a, b, and c.

 sin(a  1) x  sin x
 , if x  0
 x
f (x) =  c, if x  0 . [CBSE 2008]

 x  bx  x , if x  0
2

 bx 3 2

Q.7 Show that the function f (x) is defined by

 sin x
  cos x, x  0
 x
f (x) =  2, x0 is continuous at x = 0. [CBSE 2009]

4 1 1 x

, x0
 x

Q.8 Find all points of discontinuity of f, where f is defined as follows:


| x | 3, x  3

f (x) =   2x ,  3  x  3 [CBSE 2010]
6x  2, x  3

5 ; x2

Q.9 Find the values of a and b such that the function f (x) =  ax  b; 2  x  10
21; x  10
is a continuous function [CBSE 2011]

BANSAL CLASSES Private Ltd. ‘Bansal Tower’, A-10, Road No.-1, I.P.I.A., Kota-05 Page # 63
LIMIT, CONTINUITY AND DIFFERENTIABILITY OF FUNCTION

 1  kx  1  kx
 , if  1  x  0
Q.10 Find the value of k, for which f ( x )   x
 2x  1
 , if 0  x  1
is continuous at x = 0 x  1 [CBSE 2013]

Q.11 Discuss the continuityand differentiabilityof the function


f(x) = |x| + |x – 1| in the interval (–1, 2) [CBSE 2015]

 sin(a  1) x  2 sin x
 , x0
 x
Q.12 If f (x) =  2 , x0
 1  bx  1
 , x0
 x
is continuous at x = 0, then find the values of a and b. [CBSE 2016]
Q.13 Determine the value of 'k' for which the following function is continuous at x = 3.

 ( x  3)2  36
 , x3
f (x) =  x 3 [CBSE 2017]

 k , x 3
SECTION-B
(Potential Problems Based on CBSE)
3x 2  ax  a  3
Q.1 If the function f (x) = is continuous at x = – 2. Find f (–2).
x2  x  2
 1  sin 3 x 
 2
if x
 3 cos x 2
 
Q.2 Determine a & b so that f is continuous at x = where f(x) =  a if x
2  2
 b(1  sin x ) 
 if x
   2 x 
2
2

(e 2 x  1)  ( x  1)(e x  e x )
 , if x  0
x ( e x
 1)
Q.3 Let f (x) = 

k, if x  0
If f (x) is continuous at x = 0 then find the value of k.
1  cos 4x
if x  0
 x2
Q.4 If f (x) =  a if x  0
 x
if x  0
16  x  4
Find the value of 'a' if possible so that the function is continuous at x = 0.
BANSAL CLASSES Private Ltd. ‘Bansal Tower’, A-10, Road No.-1, I.P.I.A., Kota-05 Page # 64
LIMIT, CONTINUITY AND DIFFERENTIABILITY OF FUNCTION

( x 2  3x  1) tan x
 if x  0
x 2  2x
Q.5 If f (x) =
 k if x  0
is continuous at x = 0, then find the value of k.
Q.6 If f and g are continuous functions with f (5) = 5 and Lim 2f ( x )  g( x )  = 6 then find the value of g(5).
x 5

sin x  sin 5x 
if x  
Q.7 f (x) =  cos x  cos 5x 4 . Find k if f is continuous at x =   .
 k if x  
 4
4
8 x  4 x  2 x  12
 if x  0
Q.8 If f (x) =  x2 is continuous at x = 0 then find the value of k.
 e x sin x  4 x  k ln 4, x  0

  4 sin x  cos x for x   2



Q.9 If f(x) =  a sin x  b for  2  x  2 is continuous then find a and b.
 cos x  2 for x  2


 1 3x 2  1
4
   x  1
Q.10 Show that the function f(x) = 5  4x 1 x  4
4  x 4x

is continuous at x = 1 and discontinuous at x = 4.

 3x  4 x  1 for x  1
2

Q.11 Let g (x) =  .


 ax  b for x  1
If g (x) is continuous and differentiable for all numbers in its domain then find a and b.

Q.12 Let f(x) = | x |3 find whether f "(x) exists  real x.

Q.13 Discuss the continuity & differentiability of the functions f (x) = x+ x  1+ x  2x R.
Also given an example of the function which is continuous everywhere but not derivable at exactly two
points.
 1 for x 0

Q.14 A function f is defined as follows : f(x) = 1 | sin x | for 0 x 2
  2
 2 x  2   for   x 
2

Discuss the continuity & differentiability at x = 0 & x = /2.

a x  2 , 0  x  2
Q.15 Let g (x) =  . If g (x) is derivable on (0, 5), then find (2a + b).
b x  2 , 2  x  5

BANSAL CLASSES Private Ltd. ‘Bansal Tower’, A-10, Road No.-1, I.P.I.A., Kota-05 Page # 65
LIMIT, CONTINUITY AND DIFFERENTIABILITY OF FUNCTION

EXERCISE-5 (Rank Booster)

n
  
Q.1 If l = Lim
n 
  (r  1) sin r  1  r sin r  then find { l }. (where { } denotes the fractional part function)
r 2

Q.2 Let graph of a function f (x) is shown in the adjacent figure. Four limits l1, l2, l3, l4 are as follows.
  sin( x  1)  
l1 = Lim f    f (x)
x 1   x  1   3

sin [f ( x )]  2 
l2 = Lim 2
x 3 x 3
  tan(2x  4)   1
l3 = Lim f   
x 2   x  2  
x
l4 = Lim f  f ( x ) 
O 1 2 3 4
x 0

p
If l2 + l3 + l4 ÷ l1 =, p, q  N then find the least value of (p + q).
q
[Note: [y] denotes greatest integer function less than or equal to y.]

sin 1 (1  {x}).cos 1 (1  {x})


Q.3 Let f(x) = then find Lim f(x) and Lim f(x), where {x} denotes the fractional
2{x} . (1  {x}) x0  x0

part function.

1
   x
 tan  x    e 2
p
If Lim   
4
Q.4 is equal to   e 2 , (p, q  N), then find the minimum value of (p + q).
x 0 x 2
q

 
 ln ax 
Q.5 If Lim ln x ln a  ln  = 6, then find the value of a.
x 0  ln x 
 
 a 

x2 x2 x2
Q.6 Let yn(x) = x2 +  ............ and y (x) = Lim y n ( x )
1  x 2 (1  x 2 )2 (1  x 2 ) n 1 n 

Discuss the continuity of yn(x) (n  N) and y(x) at x = 0

BANSAL CLASSES Private Ltd. ‘Bansal Tower’, A-10, Road No.-1, I.P.I.A., Kota-05 Page # 66
LIMIT, CONTINUITY AND DIFFERENTIABILITY OF FUNCTION

 2  sin 1  {x} · sin 1  {x}


1 2 1
for x  0
2 {x}  {x} 
 3
Q.7 Let f(x) =  where {x} is the fractional part of x.
 
 for x  0
2
Consider another function g(x) ; such that
g(x) = f(x) for x  0
= 2 2 f(x) for x < 0
Discuss the continuity of the functions f(x) & g(x) at x = 0.

Q.8 Find the number of ordered pair(s) (a, b) for which the function
 
f(x) = sgn ( x 2  ax  1) ( bx 2  2bx  1) is discontinuous at exactly one point (where a, b are integer).
[Note : sgn (x) denotes signum function of x.]

Q.9
(a) For any real number x, let [x] denote the largest integer less than or equal to x. Let f be a real valued
 x  [  x ] if [ x ] is even
function defined on the interval [– 3, 3] by f (x) = x  [ x ] if [ x ] is odd .

If L denotes the number of points of discontinuityand M denotes the number of points of non-derivability
of f (x), then find (L + M).
Let g (x) =   | x |2 · e5  |x| . If N1 denotes the number of values of  where g (x) is discontinuous
2
(b)
and N2 denotes the number of values of  where g (x) is differentiable  x  R, then find the value of
(N1 + N2).


2
 x cos 2x if x  0
Q.10 Consider the function, f (x) = 
 0 if x  0
(a) Show that f ' (0) exists and find its value
(b) Show that f ' 1 3 does not exist.
(c) For what values of x, f ' (x) fails to exist.

BANSAL CLASSES Private Ltd. ‘Bansal Tower’, A-10, Road No.-1, I.P.I.A., Kota-05 Page # 67
LIMIT, CONTINUITY AND DIFFERENTIABILITY OF FUNCTION

EXERCISE-1
SPECIAL DPP-1 [LIMIT]

Q.1 (i) 1 ; (ii) – 1 ; (iii) does not exist ; (iv) 1


Q.2 (a) – 1 ; (b) – 2 ; (c) does not exist ; (d) 2 ; (e) 0 ; (f) does not exist ; (g) 1 ; (h) 3
Q.3 D.N.E. Q.4 (a) (i) 4 (ii) – 2 ; (b) (i) 0 (ii) 0
Q.5 a = 15; l = – 1 Q.6 (a) 0 ; (b) ± 
2
Q.7 (a) False ; (b) False ; (c) True ; (d) True ; (e) False Q.8
3
1 ab
Q.9 100 Q.10 Q.11
4 2
Q12 For a > 1 the limit equals 1 if x  – , and 0 if x  – . For a < 1 the limit equals 0 if x  + ,
1
and 1 if x  – . For a = 1 the limit is equal to .
2
Q.13 b, c Q.14 B Q.15 B Q.16 A Q.17 C
Q.18 C Q.19 A Q.20 A, B, C, D Q.21 A, B Q.22 A, B, D

SPECIAL DPP-2 [LIMIT]


1 1 2
Q.1 (a) 1; (b) 4; (c) ; (d) Q.2 2 Q.3 Q.4 –2
2 3 2
1
Q.5 Q.6 4 Q.7 A Q.8 C Q.9 C
2
Q.10 D Q.11 B Q.12 D Q.13 A, B, D Q.14 A, D
Q.15 0009

SPECIAL DPP-3 [LIMIT]


sin x 1 1 
Q.1 Q.2 Q.3 Q.4
x 2 2 
Q.5 (a) exist and is equal to 1; (b) DNE; RHL = 4 and LHL = 0
Q.6 D Q.7 B Q.8 D Q.9 D Q.10 C
Q.11 A Q.12 A, C Q.13 30 Q.14 96

SPECIAL DPP-4 [LIMIT]


Q.1 C Q.2 A Q.3 B Q.4 D Q.5 C
Q.6 A Q.7 C Q.8 D Q.9 A Q.10 C
Q.11 C Q.12 A, B Q.13 (A) Q; (B) T; (C) P; (D) R; (E) S Q.14 100

SPECIAL DPP-5 [LIMIT]


Q.1 C Q.2 C Q.3 B Q.4 B Q.5 A
Q.6 C Q.7 A Q.8 B Q.9 A, C Q.10 A, C
Q.11 (A) R; (B) R ; (C) P; (D) T Q.12 1 Q.13 5 Q.14 5

BANSAL CLASSES Private Ltd. ‘Bansal Tower’, A-10, Road No.-1, I.P.I.A., Kota-05 Page # 68
LIMIT, CONTINUITY AND DIFFERENTIABILITY OF FUNCTION

SPECIAL DPP-6 [CONTINUITY]


1 3 2
Q.1 (a) (ln 3)2 ; (b) ; (c) 1; (d)  1/2ab Q.2 Q.3 C
64 32
Q.4 D Q.5 A Q.6 D Q.7 C Q.8 C
Q.9 D Q.10 B Q.11 B Q.12 B Q.13 C, D
Q.14 A, D Q.15 A, B, C, D

SPECIAL DPP-7 [CONTINUITY]


Q.1 D Q.2 B Q.3 B Q.4 A
Q.5 D Q.6 A Q.7 D Q.8 D
Q.9 A, B, C Q.10 B, C, D Q.11 A, B, D Q.12 A, B, C, D
Q.13 A, C, D Q.14 A, B, D Q.15 5

SPECIAL DPP-8 [DERIVABILITY]


Q.1 (i) False; (ii) True; (iii) True (iv) True Q.2 C Q.3 A Q.4 C
Q.5 A Q.6 D Q.7 D Q.8 B
Q.9 C Q.10 D Q.11 A, C Q.12 A, B Q.13 A, C
Q.14 A, B, C Q.15 (A) R, S; (B) P, S; (C) P, Q; (D) P, S

SPECIAL DPP-9 [DERIVABILITY]


Q.1 C Q.2 A Q.3 C Q.4 D Q.5 A
Q.6 B Q.7 C Q.8 A, C Q.9 (A) R, (B) P, (C) Q, (D) R
Q.10 (A) R ; (B) T ; (C) S ; (D) R Q.11 7, where a = 3 ; b = – 4 Q.12 4

SPECIAL DPP-10 [DERIVABILITY]


Q.1 D Q.2 A Q.3 D Q.4 B Q.5 A
Q.6 C Q.7 B Q.8 D Q.9 B Q.10 B, D
Q.11 A, B, C, D Q.12 A, C, D Q.13 A, B, D Q.14 (A) T, (B) R, (C) P, (D) Q
Q.15 74
EXERCISE-2
SECTION-A [LIMIT]

1 8 1 3
Q.1 a = 2; r = ;S= Q.2 Q.3
4 3 32 2
1
Q.4 Q.5 2
2
1
Q.6 (i) a =1, b = 1 (ii) a = 1 , b = Q.7 27 Q.8 72
2
Q.9 16 Q.10 36 Q.11 8
1

Q.12 (a) 7; (b) 15 Q.13 (i) e–1 ; (ii) e–1/2
; (iii) c = ln2 ; (iv) e 2

Q.14 e Q.15 (i) – 14; (ii) 5


Q.16 9 Q.17 e
Q.18 2 Q.19 (a) 2, (b) D.N.E., (c) 0, (d) 0
Q.20 (a) 2; (b) 1/2

BANSAL CLASSES Private Ltd. ‘Bansal Tower’, A-10, Road No.-1, I.P.I.A., Kota-05 Page # 69
LIMIT, CONTINUITY AND DIFFERENTIABILITY OF FUNCTION

SECTION-B [CONTINUITY]

Q.1 a = 0, b = 1 Q.2 a = 0 ; b = 1 Q.3 30


Q.4 5 Q.5 5 Q.6 c = 1, a, b  R
1
Q.7 2 Q.8 a= Q.9 e2 + e–2
e
Q.10 4 Q.11 6

SECTION-C [DERIVABILITY]
Q.1 0<n1 Q.2 2 Q.3 5
1 3
Q.4 f is cont. but not diff. at x = 0 Q.5 a= ,b= Q.6 5
2 2
Q.7 not derivable at x = 0 & x = 1
Q.8 discontinuous & not derivable at x = 1, continuous but not derivable at x = 2
Q.9 5 Q.10 3
Q.11(a) 9; (b) 99 Q.12 (a) 2; (b) 2

EXERCISE-3
SECTION-A
LIMIT
Q.1 4 Q.2 3 Q.3 1 Q.4 1 Q.5 4
Q.6 2 Q.7 4 Q.8 2 Q.9 1 Q.10 1
Q.11 1 Q.12 3 Q.13 1 Q.14 1 Q.15 4
Q.16 2 Q.17 1
CONTINUITY & DIFFERENTIABILITY
Q.1 3 Q.2 3 Q.3 2 Q.4 3 Q.5 2
Q.6 4 Q.7 3 Q.8 2 Q.9 3 Q.10 3
Q.11 3 Q.12 3 Q.13 3

SECTION-B
Q.1 B Q.2 A Q.3 D Q.4 D Q.6 C
Q.7 D Q.8 C Q.9 a = 1; b = 0(gof)'(0) = 0 Q.10 C
2
Q.11 f (a–) = 0 Q.12 1 Q.13 (a) A, (b) y – 2 = 0 Q.14 A, C

Q.15 C Q.16 A, C Q.17 (a) D; (b) A, B, C, D; (c) BC; (d) A
Q.18 (a) B, (b) B, (c) B, (d) BD Q.19 AD Q.20 3
Q.21 0 Q.22 D Q.23 AD Q.24 ABC Q.25 2
Q.26 A Q.27 7 Q.28 BC Q.29 AB Q.30 BCD
Q.31 BC Q.32 2.00 Q.33 D

BANSAL CLASSES Private Ltd. ‘Bansal Tower’, A-10, Road No.-1, I.P.I.A., Kota-05 Page # 70
LIMIT, CONTINUITY AND DIFFERENTIABILITY OF FUNCTION

EXERCISE-4
SECTION-A
3b  2 2
Q.2 Discontinuous Q.3 not continuous Q.4 a=  b
3 3
Q.5 f (x) cannot be continuous at x = 0 for any value of m.

3 1
Q.6 a= ; b = any real value except zero; c =
2 2
Q.8 only point of discontinuity of f(x) is x = 3.
Q.9 a = 2; b = 1 Q.10 k = –1
Q.11 Continuous everywhere in (–1, 2) but not differentiable at x = 0 and 1
Q.12 a = –1, b = 4 Q.13 12

SECTION-B
Q.1 –1 Q.2 a = 1/2, b = 4 Q.3 1 Q.4 8
Q.5 1 2 Q.6 4 Q.7 1 Q.8 ln 2
Q.9 a =  1, b = 3 Q.11 a = 4 and b = – 4 Q.12 Yes
Q.13 continuous  x  R, but not derivable at x = 0,1 & 2

Q.14 continuous but not differentiable at x = 0 ; differentiable & continuous at x = .
2
Q.15 3

EXERCISE-5
 
Q.1 –3 Q.2 7 Q.3 ,
2 2 2
Q.4 7 Q.5 e3

Q.6 yn(x) is continuous at x = 0 for all n and y(x) is dicontinuous at x = 0


 
Q.7 f(0+) = ; f(0) =  f is discont. at x = 0 ; g(0+) = g(0) = g(0) = /2  g is cont. at x = 0
2 4 2
Q.8 6 Q.9 (a) 8; (b) 2
 1    1   1
Q.10 (a) f ' (0) = 0, (b) f '   = – and f '   = , (c) x =
3 2 nI
3 2   2n  1

BANSAL CLASSES Private Ltd. ‘Bansal Tower’, A-10, Road No.-1, I.P.I.A., Kota-05 Page # 71
in Last Nineteen Years

*CJAEMAT03*
CJAEMAT03

You might also like